Você está na página 1de 86

8/5/2018 Taller de Autoaprendizaje No 1a

Taller de Autoaprendizaje No 1a
Due: 11:59pm on Sunday, April 29, 2018
You will receive no credit for items you complete after the assignment is due. Grading Policy

Dimensions of Physical Quantities

Learning Goal:
To introduce the idea of physical dimensions and to learn how to find them.
Physical quantities are generally not purely numerical: They have a particular dimension or combination of dimensions
associated with them. Thus, your height is not 74, but rather 74 inches, often expressed as 6 feet 2 inches. Although feet and
inches are different units they have the same dimension--length.

Part A
In classical mechanics there are three base dimensions. Length is one of them. What are the other two?

Hint 1. MKS system


The current system of units is called the International System (abbreviated SI from the French Système International).
In the past this system was called the mks system for its base units: meter, kilogram, and second. What are the
dimensions of these quantities?

ANSWER:

acceleration and mass

acceleration and time

acceleration and charge

mass and time

mass and charge

time and charge

Correct

There are three dimensions used in mechanics: length ( l), mass ( m), and time ( t). A combination of these three dimensions
suffices to express any physical quantity, because when a new physical quantity is needed (e.g., velocity), it always obeys an
equation that permits it to be expressed in terms of the units used for these three dimensions. One then derives a unit to
measure the new physical quantity from that equation, and often its unit is given a special name. Such new dimensions are called
derived dimensions and the units they are measured in are called derived units.
2
For example, area A has derived dimensions [A] = l . (Note that "dimensions of variable x" is symbolized as [x] .) You can find
these dimensions by looking at the formula for the area of a square A = s2 , where s is the length of a side of the square. Clearly
2 2
[s] = l . Plugging this into the equation gives [A] = [s] = l .

Part B

Find the dimensions [V ] of volume.

https://session.masteringphysics.com/myct/assignmentPrintView?assignmentID=6248360 1/86
8/5/2018 Taller de Autoaprendizaje No 1a
Express your answer as powers of length ( l), mass ( m), and time ( t).

Hint 1. Equation for volume


You have likely learned many formulas for the volume of various shapes in geometry. Any of these equations will give
you the dimensions for volume. You can find the dimensions most easily from the volume of a cube V = e3 , where e
is the length of the edge of the cube.

ANSWER:

[V ] = l
3

Correct

Part C
Find the dimensions [v] of speed.

Express your answer as powers of length ( l), mass ( m), and time ( t).

Hint 1. Equation for speed


Speed v is defined in terms of distance d and time t as

d
v= .
t

Therefore, [v] .
= [d]/[t]

Hint 2. Familiar units for speed


You are probably accustomed to hearing speeds in miles per hour (or possibly kilometers per hour). Think about the
dimensions for miles and hours. If you divide the dimensions for miles by the dimensions for hours, you will have the
dimensions for speed.

ANSWER:

l
[v] =
t

Correct

The dimensions of a quantity are not changed by addition or subtraction of another quantity with the same dimensions. This
means that Δv , which comes from subtracting two speeds, has the same dimensions as speed.

It does not make physical sense to add or subtract two quanitites that have different dimensions, like length plus time. You can
add quantities that have different units, like miles per hour and kilometers per hour, as long as you convert both quantities to the
same set of units before you actually compute the sum. You can use this rule to check your answers to any physics problem you
work. If the answer involves the sum or difference of two quantities with different dimensions, then it must be incorrect.

This rule also ensures that the dimensions of any physical quantity will never involve sums or differences of the base dimensions.
(As in the preceeding example, l + t is not a valid dimension for a physical quantitiy.) A valid dimension will only involve the
2
product or ratio of powers of the base dimensions (e.g. m2/3 l t−2 ).

https://session.masteringphysics.com/myct/assignmentPrintView?assignmentID=6248360 2/86
8/5/2018 Taller de Autoaprendizaje No 1a

Part D
Find the dimensions [a] of acceleration.

Express your answer as powers of length ( l), mass ( m), and time ( t).

Hint 1. Equation for acceleration


In physics, acceleration a is defined as the change in velocity in a certain time. This is shown by the equation
a = Δv/Δt. The Δ is a symbol that means "the change in."

ANSWER:

l
[a] = 2
t

Correct

± PSS 1.2 Unit Conversions

Learning Goal:

To practice Problem-Solving Strategy 1.2 Unit Conversions.


A gallon of water in the United States weighs about 8.33 lb . In other words, the density of water is 8.33 lb/gal . What is the
density of water in kg/m3 ? What is the density of water in g/cm3 ?

Problem-Solving Strategy: Unit conversions

IDENTIFY the relevant concepts:


In most cases, you're best off using the fundamental SI units (meters, kilograms, seconds) within a problem. If you need the
answer to be in a different set of units, wait until the end of the problem to make the conversion.

SET UP the problem and EXECUTE the solution as follows:


Units are multiplied and divided just like ordinary algebraic symbols. This gives us an easy way to convert a quantity from one set
of units to another. The key idea is to express the same physical quantity in two different units and form an equality. For example,
since 1 min = 60 sec the ratio (1 min)/(60 sec) equals 1.

EVALUATE your answer:


If you do your units conversion correctly, unwanted units will cancel. Finally, check whether your answer is reasonable. If you
have converted to a smaller unit, for example, the number representing the quantity should be larger.

IDENTIFY the relevant concepts


The physical property of density is given by mass/volume. The SI unit for mass is the kilogram (kg) and for volume it is the cubic
meter (m3 ). Therefore, density should be given in units of kg/m3 .

SET UP the problem and EXECUTE the solution as follows

Part A

Calculate the density of water in kg/m3 .


Express your answer in kilograms per cubic meter using three significant figures.

https://session.masteringphysics.com/myct/assignmentPrintView?assignmentID=6248360 3/86
8/5/2018 Taller de Autoaprendizaje No 1a

Hint 1. Find the conversion factor between pounds and kilograms


Which of the following expressions is the correct conversion factor needed when converting pounds into kilograms?
ANSWER:

1.00 lb

2.20 kg

1.00 kg

2.20 lb

2.20 lb

1.00 kg

Hint 2. Find the conversion factor between gallons and cubic meters
Which of the following expressions is the correct conversion factor needed when converting gallons into cubic
meters?

ANSWER:

264 gal

1.00 m3

3
264 m

1.00 gal

3
1.00 m

264 gal

ANSWER:

8.33 lb/gal = 1000 kg/m


3

Correct

If you were to complete the conversion and find the equivalent density in kg/m3 of 8.33 lb/gal , you would write

lb 264 gal
lb 1.00 kg kg
8.33 = 8.33 × × = 1000 .
gal gal 2.20 lb 1.00 m3 m3

Part B
Now, convert the density of water to g/cm3 .

Express your answer in grams per cubic centimeter using three significant figures.

Hint 1. Find the conversion factor between cubic meters and cubic centimeters
Which of the following is the correct relationship between cubic meters and cubic centimeters?

https://session.masteringphysics.com/myct/assignmentPrintView?assignmentID=6248360 4/86
8/5/2018 Taller de Autoaprendizaje No 1a
ANSWER:

3 6 3
1 m = 10 cm

3 4 3
1 m = 10 cm

3 2 3
1 m = 10 cm

3 −2 3
1 m = 10 cm

3 −4 3
1 m = 10 cm

3 −6 3
1 m = 10 cm

ANSWER:

1000 kg/m3 = 1.00 g/cm


3

Correct

If you had 1 cm3 of water (about the size of a sugar cube), it would have a mass of 1 g .

EVALUATE your answer

Part C
The same physical quantity, such as density, can be reported using different units. Above, you found that water has a density
of 1000 kg/m3 = 1 g/cm3 . Because the density of water must be the same regardless of what units you use to measure
it, you can conclude that an object whose density is 1 kg/m3 must be less dense than water. In other words, 1 kg/m3 is
less than 1 g/cm3 .

If you had three different objects with densities of 1 kg/m3 , 1 g/m3 , and 1 kg/mm3 , which object would be the most
dense?

Rank the given densities from most to least dense. To rank items as equivalent, overlap them.

Hint 1. How to approach the problem


If all these densities were given in the same units, you could easily compare the objects to identify the most dense
and least dense. Since the units given are all different, convert all the densities to a common set of units, such as
kg/m , before making your comparison.
3

ANSWER:

https://session.masteringphysics.com/myct/assignmentPrintView?assignmentID=6248360 5/86
8/5/2018 Taller de Autoaprendizaje No 1a

Reset Help

Most dense Least dense

Correct
These three densities are readily compared if they are converted to the SI unit of density, kg/m3 :

3 9 3
1 kg/mm = 10  kg/m

3 3
1 kg/m = 1 kg/m

3 −3 3
1 g/m = 10  kg/m

Converting Units: The Magic of 1

Learning Goal:

To learn how to change units of physical quantities.


Quantities with physical dimensions like length or time must be measured with respect to a unit, a standard for quantities with this
dimension. For example, length can be measured in units of meters or feet, time in seconds or years, and velocity in meters per
second.

When solving problems in physics, it is necessary to use a consistent system of units such as the International System
(abbreviated SI, for the French Système International) or the more cumbersome English system. In the SI system, which is the
preferred system in physics, mass is measured in kilograms, time in seconds, and length in meters. The necessity of using
consistent units in a problem often forces you to convert some units from the given system into the system that you want to use
for the problem.

The key to unit conversion is to multiply (or divide) by a ratio of different units that equals one. This works because multiplying
any quantity by one doesn't change it. To illustrate with length, if you know that 1 inch = 2.54 cm, you can write

2.54 cm
1= .
1 inch

https://session.masteringphysics.com/myct/assignmentPrintView?assignmentID=6248360 6/86
8/5/2018 Taller de Autoaprendizaje No 1a
To convert inches to centimeters, you can multiply the number of inches times this fraction (since it equals one), cancel the inch
unit in the denominator with the inch unit in the given length, and come up with a value for the length in centimeters. To convert
centimeters to inches, you can divide by this ratio and cancel the centimeters.

For all parts, notice that the units are already written after the answer box; don't try to write them in your answer also.

Part A
How many centimeters are there in a length 54.7 inches ?

Express your answer in centimeters to three significant figures.


ANSWER:

139 cm

Correct

Sometimes you will need to change units twice to get the final unit that you want. Suppose that you know how to convert from
centimeters to inches and from inches to feet. By doing both, in order, you can convert from centimeters to feet.

Part B
Suppose that a particular artillery piece has a range R = 4840 yards . Find its range in miles. Use the facts that
1 mile = 5280 f t and 3 f t = 1 yard.

Express your answer in miles to three significant figures.

Hint 1. Convert yards to feet


The first step in this problem is to convert from yards to feet, because you know how to then convert feet into miles.
Convert 4840 yards into feet. Use

3 ft
1= .
1 yard

Express your answer in feet to three significant figures.

ANSWER:

1.45×104 ft

ANSWER:

4840 yards = 2.75 miles

Correct

Often speed is given in miles per hour (mph), but in physics you will almost always work in SI units. Therefore, you must convert
mph to meters per second (m/s).

https://session.masteringphysics.com/myct/assignmentPrintView?assignmentID=6248360 7/86
8/5/2018 Taller de Autoaprendizaje No 1a

Part C
What is the speed of a car going v = 1.000 mph in SI units? Notice that you will need to change from miles to meters and
from hours to seconds. You can do each conversion separately. Use the facts that 1 mile = 1609 m and
1 hour = 3600 s.

Express your answer in meters per second to four significant figures.

Hint 1. Convert miles to meters


In converting 1.000 mph into meters per second, you will need to multiply by

1609 m
1= .
1 mile
When you do this, the miles will cancel to leave you with a value in meters per hour. You can then finish the
conversion. What is v = 1.000 mph in meters per hour?
Express your answer in meters per hour to four significant figures.

ANSWER:

v = 1609 m/hour

Hint 2. Convert hours to seconds


Which of the following would you multiply 1609 m/hours by to convert it into meters per second (m/s)?

ANSWER:

3600 s

1 hour

3600 s

1 hour

1 hour

3600 s

ANSWER:

v = 0.4469 m/s

Correct
Notice that by equating the two values for v , you get 1.000 mph = 0.4469 m/s. It might be valuable to
remember this, as you may frequently need to convert from miles per hour into more useful SI units. By
remembering this relationship in the future, you can reduce this task to a single conversion.

± A Trip to Europe

Learning Goal:

To understand how to use unit conversion to solve problems.


https://session.masteringphysics.com/myct/assignmentPrintView?assignmentID=6248360 8/86
8/5/2018 Taller de Autoaprendizaje No 1a
Unit conversion is not limited to physics problems but is part of our everyday life, correct use of conversion factors is essential to
working through problems of practical importance.
For example, unit conversion could be used in problems involving currency exchange. Say you want to calculate how many
euros you get if you exchange 3600 US dollars (USD), given the exchange rate 1 EUR = 1.20 USD , that is, 1 euro to 1.20
US dollars. Begin by writing down the starting value, 3600 USD. This can also be written as a fraction:

3600 USD
.
1
Next, convert dollars to euros. This conversion involves multiplying by a simple conversion factor derived from the exchange rate:

1.00 EUR
.
1.20 USD

Note that the "dollar" unit, USD, should appear on the bottom of this conversion factor, since USD appears on the top of the
starting value.

Finally, since dollars are divided by dollars, the units can be canceled and the final result is

3600 USD 1.00 EUR


( )( ) = 3000 EUR .
1 1.20 USD

Currency exchange is only one example of many practical situations where unit conversion may help you to work through
problems. Remember that unit conversion involves multiplying a given value by a conversion factor, resulting in a value in the
new units. The conversion factor can be the ratio of any two quantities, as long as the ratio is equal to one.

You and your friends are organizing a trip to Europe. Your plan is to rent a car and drive through the major European capitals. By
consulting a map you estimate that you will cover a total distance of 5000 km. Consider the euro-dollar exchange rate given in
the introduction and use unit conversion to work through these simple problems.

Part A
You select a rental package that includes a car with an average consumption of 6.00 liters of fuel per 100 km. Considering
that in Europe the average fuel cost is 1.063 euros/liter, how much (in US dollars) will you spend in fuel on your trip?

Express your answer numerically in US dollars to three significant figures.

Hint 1. How to approach the problem


Begin with writing the total distance in kilometers. Then multiply this by the consumption rate [in liters/(100 km)] of
your rental car to calculate how many liters of fuel are needed. Then compute the total cost of fuel (in euros) by
multiplying your expression by the average fuel cost. Finally, convert the total cost of fuel to dollars.

Hint 2. Find the unit factor to express the cost of fuel in euros
Which of these unit factors can be used to calculate the cost of fuel in euros?

Hint 1. Conversion factor


When calculating the total cost of fuel in euros, you divide kilometers by kilometers (and cancel out the
kilometer units) and multiply the resulting value in liters by a conversion factor that has the liter unit at the
bottom, so that again you can cancel out units.

ANSWER:

https://session.masteringphysics.com/myct/assignmentPrintView?assignmentID=6248360 9/86
8/5/2018 Taller de Autoaprendizaje No 1a

1 EUR

1.063 liters

1.063 EUR

1 liter

1 liter

1.063 EUR

1.063 liters

1 EUR

Correct

Hint 3. Find the unit factor to convert euros to dollars


Which of these unit factors can be used to convert the total cost of fuel to dollars?

Hint 1. Conversion factor


Since you are converting the cost of fuel from euros to dollars, you need to multiply by a conversion factor that
has the euro unit, EUR, on the bottom, so that you can cancel out units.

ANSWER:

1.20 USD

1 EUR

1.20 EUR

1 USD

1 USD

1.20 EUR

1 EUR

1.20 USD

Correct

Hint 4. Canceling units


You can avoid many mistakes in unit conversion problems if you are careful to cancel units.

ANSWER:

Cost of fuel = 383 USD

Answer Requested

Part B

https://session.masteringphysics.com/myct/assignmentPrintView?assignmentID=6248360 10/86
8/5/2018 Taller de Autoaprendizaje No 1a
How many gallons of fuel would the rental car consume per mile?

Express your answer numerically in gallons per mile to three significant figures.

Hint 1. How to approach the problem


Begin with writing the consumption rate in liters per kilometer. Then multiply this by the unit factor needed to convert
kilometers to miles. Finally convert liters to gallons. Be sure to cancel units so that your answer is dimensionally
consistent.

Hint 2. Find the unit factor to convert kilometers to miles


Which of these unit factors can be used to convert kilometers to miles?

Hint 1. Conversion factor


Since you are converting the car's fuel consumption per kilometer to the car's fuel consumption per mile you
need to multiply by a conversion factor that has the kilometer unit on the top, so that you can cancel out units.

ANSWER:

1 km

1.609 miles

1.609 km

1 mile

1 mile

1.609 km

1.609 miles

1 km

Hint 3. Find the unit factor to convert liters to gallons


Which of these unit factors can be used to convert liters to gallons?

Hint 1. Conversion factor


Since you are converting the car's consumption in liters to gallons, you need to multiply by a conversion factor
that has the liter unit at the bottom, so that you can cancel out units.

ANSWER:

https://session.masteringphysics.com/myct/assignmentPrintView?assignmentID=6248360 11/86
8/5/2018 Taller de Autoaprendizaje No 1a

3.785 gallons

1 liter

1 gallon

3.785 liters

1 liter

3.785 gallons

3.785 liters

1 gallons

ANSWER:

2.55×10−2 gallons/mile

Correct

Part C
What is the average cost, in dollars per gallon, of fuel in Europe?

Express your answer numerically in dollars per gallon to three significant figures.

Hint 1. How to approach the problem


Begin with writing the cost of fuel in euros per liter. Then multiply this by the unit factor needed to convert euros to
dollars. Finally convert liters to gallons. Be sure to cancel units, to make sure that your answer is dimensionally
consistent.

Hint 2. Find the unit factor to convert euros to dollars


Which of these unit factors can be used to convert the price of fuel from euros to dollars?

ANSWER:

1.20 EUR

1 USD

1 USD

1.20 EUR

1.20 USD

1 EUR

1 EUR

1.20 USD

Hint 3. Find the unit factor to convert liters to gallons


Which of these unit factors can be used to convert the price of fuel per liter to a price per gallon?
ANSWER:

https://session.masteringphysics.com/myct/assignmentPrintView?assignmentID=6248360 12/86
8/5/2018 Taller de Autoaprendizaje No 1a

3.785 gallons

1 liter

1 gallon

3.785 liters

1 liter

3.785 gallons

3.785 liters

1 gallon

ANSWER:

4.83 USD/gallon

Correct

± Moving at the Speed of Light

Part A
How many nanoseconds does it take light to travel a distance of 5.00 km in vacuum?
Express your answer numerically in nanoseconds.

Hint 1. How to approach the problem


Light travels at a constant speed; therefore, you can use the formula for the distance traveled in a certain amount of
time by an object moving at constant speed. Before performing any calculations, it is often recommended, although it
is not strictly necessary, to convert all quantities to their fundamental units rather than to multiples of the fundamental
unit.

Hint 2. Find how many seconds it takes light to travel the given distance
8
Given that the speed of light in vacuum is 3.00 × 10 m/s, how many seconds does it take light to travel a distance
of 5.00 km ?
Express your answer numerically in seconds.

Hint 1. Find the time it takes light to travel a certain distance


How long does it take light to travel a distance r? Let c be the speed of light.

Hint 1. The speed of an object


The equation that relates the distance s traveled by an object with constant speed v in a time t is

s = vt .

https://session.masteringphysics.com/myct/assignmentPrintView?assignmentID=6248360 13/86
8/5/2018 Taller de Autoaprendizaje No 1a
ANSWER:

r⋅c

r
c

c
r

Hint 2. Convert the given distance to meters


Convert d = 5.00 km to meters.
Express your answer numerically in meters.

Hint 1. Conversion of kilometers to meters

Recall that 1 km = 10
3
m .

ANSWER:

5.00 km = 5000 m

ANSWER:

1.67×10−5 s

Incorrect; Try Again

ANSWER:

1.67×104 ns

Correct

± Scientific Notation

A number written in scientific notation has the form a × 10 k , where 1 ≤ a < 10 and k is an integer, that is, k is one of
. . . −3, −2, −1, 0, 1, 2, 3....

Part A
Determine the values of a and k when 299, 790, 000 is written in scientific notation.
Enter a and k, separated by commas.
ANSWER:
https://session.masteringphysics.com/myct/assignmentPrintView?assignmentID=6248360 14/86
8/5/2018 Taller de Autoaprendizaje No 1a

a k , = 2.9979,8

Answer Requested

The value 2.9979 × 10 8 is the magnitude of the speed of light in meters per second, to five significant figures. This is
really fast: about 675 million miles per hour!

Part B
Determine the values of a and k when 0.51 is written in scientific notation.

Enter a and k, separated by commas.


ANSWER:

a k , = 5.1,-1

Answer Requested
The value 0.51 is the magnitude of the energy associated with a stationary electron, sometimes called its "rest energy,"
expressed in millions of electron volts, or MeV.

Part C

Now consider the expression 4.0 × 10 3 + 4 × 10


2
. Determine the values of a and k when the value of this expression is
written in scientific notation.
Enter a and k, separated by commas.

Hint 1. How to approach the problem


Write both numbers such that they have the same power of 10. In this case it would be best to choose the common
power to be either 3 or 2, since these are the two that appear in the expression given. Now you can add the factors
that are multiplied by 10 n . For example, 2 × 10 13 + 3 × 10 13 = 5 × 10 13 .

ANSWER:

a k , = 4.4,3

Answer Requested

The value 4 × 10 3 = 4000 is the approximate radius of the earth in miles.

Part D
Finally, consider the expression
−11 24
(6.67×10 )(5.97×10 )

6 2
.
(6.38×10 )

Determine the values of a and k when the value of this expression is written in scientific notation.

Enter a and k, separated by commas.


https://session.masteringphysics.com/myct/assignmentPrintView?assignmentID=6248360 15/86
8/5/2018 Taller de Autoaprendizaje No 1a

Hint 1. A walk-through
2
Express all the numbers in the expression using scientific notation. To do this, recall that (a × 10 k ) = a
2
× 10
(2⋅k)
.
Then you should get a fraction of the form
k k
(a1 ×10 1 )(a2 ×10 2 )

k
,
a3 ×10 3

or
k k
a1 ⋅a2 10 1 ⋅10 2
.
a3 k
10 3

To determine a, you have to divide a1 ⋅ a2 by a3 . This will often give you a number that is not between 1 and 10; in
that case, you have to write it in scientific notation, for instance,
a1 ⋅a2 k4

a3
= a4 × 10 .

The next step is to find k5 in the expression


k k
k5 10 1 ⋅10 2
10 = .
k
10 3

Do this by subtracting k3 from the sum of k1 and k2 :

k5 = (k1 + k2 ) − k3 .

The value of a you are looking for is equal to a4 ; k is equal to k4 + k5 .

ANSWER:

a k , = 9.78,0

Answer Requested
This is the magnitude of the acceleration experienced by a freely falling body near the surface of the earth, expressed in
meters per second squared (m/s2 ). This acceleration is usually denoted by the symbol g. You may learn later that this
2
acceleration is related to the mass Me and radius Re of the earth as g = GMe /Re , where
N ⋅ m /kg is called the gravitational constant, postulated by Sir Isaac Newton and first
−11 2 2
G = 6.67 × 10

measured by Henry Cavendish in 1798.

Significant Figures

Part A

To seven significant figures, the mass of a proton is 1.672623 × 10 −27 kg . Which of the following choices demonstrates
correct rounding?
Check all that apply.

ANSWER:

https://session.masteringphysics.com/myct/assignmentPrintView?assignmentID=6248360 16/86
8/5/2018 Taller de Autoaprendizaje No 1a

−27
1.672 × 10 kg

−27
1.67 × 10 kg

−27
1.67263 × 10 kg

Correct

The number 1.672 × 10 −27 is incorrect because when we round to four significant figures we get 1.673, not
1.672. Similarly, 1.67263 × 10 −27 is incorrect because when we round to six significant figures we get 1.67262,
not 1.67263.

Part B

To eight significant figures, Avogadro's constant is 6.0221367 × 10 23 mol


−1
. Which of the following choices demonstrates
correct rounding?
Check all that apply.

ANSWER:

23 −1
6.022 × 10 mol

23 −1
6.0 × 10 mol

23 −1
6.02214 × 10 mol

Answer Requested
All these options are correct; they represent different levels of precision, even though the numerical value is the same.

Made to Order (of Magnitude)

Learning Goal:
To be able to make order-of-magnitude calculations.
Imagine that a company wants to build a new factory. Such a complex project would involve significant investment in terms of
both time and money. Consequently, before construction can start the company asks for an estimate of the total cost. Although
estimate figures are not exact, they are still helpful: For instance, if the projected cost is three times the amount of money that the
company is willing to spend, the project will be canceled or substantially changed.

Individuals make such estimates all the time. For instance, when you need to drive somewhere for a meeting, you can roughly
predict how much time you will spend on the road and depart accordingly. Of course, the actual travel time is unlikely to be
exactly the same as the estimated one—but it still helps to make an estimate so that you can decide when to leave.

Physicists must frequently make such estimates—known as order-of-magnitude calculations—as part of their job. Depending on
the results of the estimate, a potentially lengthy and costly research project may be postponed, canceled, or redesigned. Being
able to make a quick calculation and get a "ball-park figure" of the expected result is an important skill for a scientist, involving
processes such as identifying relevant information, searching for this information, and using your experience or background
knowledge.

In this problem, you will practice making such order-of-magnitude calculations.

https://session.masteringphysics.com/myct/assignmentPrintView?assignmentID=6248360 17/86
8/5/2018 Taller de Autoaprendizaje No 1a
What is the total mass of all the people on earth?
It is impossible, of course, to give an accurate answer to this question. However, it is quite possible to find the order of magnitude
of the answer. All one needs to do is to use some common sense and, possibly, search for relevant reference information. The
calculation can proceed as follows:

There are about 7 × 10 9 people on earth. An average adult male weighs, say, 75 kg; an average adult female weighs about 60
kg, and an average child will weigh considerably less than 60 kg. Figuring roughly one child per adult, we can reasonably say

that an average person's mass is about 50 kg, which gives the total mass of all humans on our planet as

7 × 10
9
× 50 = 3.5 × 10
11
kg .

Of course, we may be off in our estimates of the average mass or number of people. While it would be unreasonable to say that
we know the total mass is 3.5 × 10 11 kg , we can be reasonably sure that we have the correct order of magnitude; that is, we
have the correct exponent to which the number 10 is raised.

In each of the following problems, you will be asked to make similar estimates.

Part A
How many people can fit into the Pentagon, which was once the largest office building in the world? Assume that everybody
must be standing on the floor.

Round the answer to the nearest power of 10 and then express your answer as the order of magnitude. For
instance, if your estimated answer is 3 × 10 5 , enter 5. If your estimated answer is 8.7 × 10 5 , you should enter 6
(rounding up to the next power of 10).

Hint 1. What reference information should you be looking for?


What information should you be looking for?
Check all that apply.
ANSWER:

What is the tallest building in the world?

What is the most massive building in the world?

What building in the world has the largest floor area?

In which country is the largest building in the world located?

How many people can be found in the largest building in the world on a typical day?

Hint 2. What numeric quantities do you need to estimate?


What numeric quantities do you need to estimate?
Check all that apply.
ANSWER:

https://session.masteringphysics.com/myct/assignmentPrintView?assignmentID=6248360 18/86
8/5/2018 Taller de Autoaprendizaje No 1a

the mass of an average person

the height of an average person

the amount of space the average person needs to work efficiently

the the area that an average person takes up while standing

the volume of an average person

ANSWER:

Correct
Your process for solving this problem might have been something like this:

First, a simple library or Internet search would tell you that the largest building in the world (in terms of the total
floor area) is the Pentagon, the main building of the U.S. Department of Defense. Its total floor area is about
2 2
f t but only 3.8 × 10 f t can be occupied; it would be hard to stand inside a wall! Assuming that an
6 6
6.5 × 10
2
average person occupies about 2.0 f t when standing (a conservative estimate), we can see that about 1.9 × 10 6
people (more than three times the entire population of Washington, DC) could fit into the Pentagon—assuming the
floors held up!

Your own answer may have been different from ours or used different details; however, the order of magnitude
was, hopefully, the same.

Part B
If the entire population of the United States forms a human chain by holding hands, how many times can such a chain be
wrapped around the earth's equator?
Round your answer to the nearest integer.
ANSWER:

Correct
Here is one way to solve this problem:

There are about 300 million people in the United States. The distance between the tips of a person's outstretched
hands is roughly equal to the height of the person. Counting children, we estimate the average palm-to-palm
distance as one meter. Since the equator is about 40 million meters long, division yields about 7.5. However, in this
part any answer between 5 and 15 is considered correct—after all, we are just estimating.

Part C
How many times does your heart beat during your lifetime?

https://session.masteringphysics.com/myct/assignmentPrintView?assignmentID=6248360 19/86
8/5/2018 Taller de Autoaprendizaje No 1a
Round the answer to the nearest power of 10 and then express your answer as the order of magnitude. For
5 5
instance, if your estimated answer is 3 × 10 , enter 5. If your estimated answer is 8.7 × 10 , you should enter 6
(rounding up to the next power of 10).
ANSWER:

Correct
On average, your heart beats about once every second. The number of seconds in the lifetime of an average U.S.
resident is
s days years 9
86, 400 × 365 × 75 = 2.4 × 10 s
day year lif etime

assuming a lifetime of 75 years. Of course, we didn't account for leap years since this is just an estimate.

Part D
Legend has it that, many centuries ago, Archimedes jumped out of his bathtub and ran across town naked screaming
"Eureka!" after he solved an especially difficult problem. Though you may not have thought of things this way before, when
you drink a glass of water, the water that you are drinking contains some water molecules that were in Archimedes'
bathwater that day, because water doesn't get created or destroyed on a large scale. It follows the water cycle, which
includes rain, evaporation, flowing of rivers into the ocean, and so on. In the more than two thousand years since his
discovery, the water molecules from Archimedes' bathwater have been through this cycle enough times that they are
probably about evenly distributed throughout all the water on the earth. When you buy a can of soda, about how many
molecules from that famous bathtub of Archimedes are there in that can?

Round the answer to the nearest power of 10 and then express your answer as the order of magnitude. For
5 5
instance, if your estimated answer is 3 × 10 , enter 5. If your estimated answer is 8.7 × 10 , you should enter 6
(rounding up to the next power of 10).

Hint 1. How to approach the problem


Assume that the water from that bathtub is evenly mixed with the water throughout the surface of the Earth--after all,
it has had more than 2000 years to do so.

Hint 2. Avogadro's number

There are about 6.0 × 10 23 molecules in 18 g of water; in other words, 1 kg of water contains about
molecules .
25
3.3 × 10

Hint 3. What to estimate


Estimate the mass of water in Archimedes' bathtub and the total mass of water on earth. (Note that most of the
earth's water is in its oceans.)

ANSWER:

https://session.masteringphysics.com/myct/assignmentPrintView?assignmentID=6248360 20/86
8/5/2018 Taller de Autoaprendizaje No 1a

Correct

We used the following assumptions: The total mass of water on the earth's surface is 1.4 × 10 21 kg (reference
information available from many different sources); the mass of the water in the bathtub is guessed to be 200 kg;
the mass of the water in the can of soda is estimated to be about 0.33 kg; and 1 kg of water contains about
molecules .
25
3.3 × 10

25
Thus the total number of molecules in the can is roughly 10 . The fraction of the bathtub molecules in the can is
) . Therefore, the number of bathtub molecules contained in the can is
21
200/(1.4 × 10

25
200×10 6
21
≈ 1.4 × 10 .
1.4×10
Your answer is most likely different but it should still have the same order of magnitude, equal to 6. In case of some
"wilder" assumptions, we count 5 and 7 as correct too.

Running and Walking

Tim and Rick both can run at speed v r and walk at speed v w , with v r > v w . They set off together on a journey of distance D.
Rick walks half of the distance and runs the other half. Tim walks half of the time and runs the other half.

Part A
How long does it take Rick to cover the distance D?
Express the time taken by Rick in terms of v r , v w , and D.

Hint 1. Compute midpoint for Rick


Find the time that it takes Rick to walk the first half of the distance, that is, to travel a distance D/2 at speed v w .

ANSWER:

D
t w,R =
2vw

Hint 2. Compute running time for Rick


Now find the time Rick spends running.
ANSWER:

D
t r,R =
2vr

Hint 3. What equation to use


Now just add the two times up and you're done.

ANSWER:

https://session.masteringphysics.com/myct/assignmentPrintView?assignmentID=6248360 21/86
8/5/2018 Taller de Autoaprendizaje No 1a

D D

tR = 2
+
2

vw vr

Correct

Part B
Find Rick's average speed for covering the distance D.
Express Rick's average speed in terms of v r and v w .

Hint 1. Calculate speed using D and time


You were given the total distance and have calculated the total time. Recall that average speed is equal to total
distance traveled divided by the amount of time it took to travel this distance.

ANSWER:

2vr vw
v ave,R =
vr +vw

Correct

Part C
How long does it take Tim to cover the distance?
Express the time taken by Tim in terms of v r , v w , and D.

Hint 1. Calculate average speed


Tim walks at speed v w half the time and runs at speed v r for the other half.
Find Tim's average speed in terms of given quantities.

Hint 1. Solve a related problem


Consider a related problem. Tim walks for a time tT /2 at speed v w and runs for a time tT /2 at speed v r .

Find the total distance he travels in time tT in terms of given quantities and tT .
ANSWER:

vw t T vr t T
dT = +
2 2

ANSWER:

https://session.masteringphysics.com/myct/assignmentPrintView?assignmentID=6248360 22/86
8/5/2018 Taller de Autoaprendizaje No 1a

vw +vr
v ave,T =
2

ANSWER:

2D
tT = vr +vw

Correct

Part D
Who covers the distance D more quickly?
Think logically, but without using the detailed answers in the previous parts.

Hint 1. Consider the relative positions at the midpoint


Imagine that both Rick and Tim do all of their walking before they start to run. Rick will start running when he has
covered half of the total distance. When Tim reaches half of the total distance, will he already have started running?

ANSWER:

Rick

Tim

Neither. They cover the distance in the same amount of time.

Correct

Part E
In terms of given quantities, by what amount of time, Δt, does Tim beat Rick?
It will help you check your answer if you simplify it algebraically and check the special case v r = vw .
Express the difference in time, Δt in terms of v r , v w , and D.
ANSWER:

2
D(vw −vr )
Δt =
2vr vw (vr +vw )

Correct

Part F

https://session.masteringphysics.com/myct/assignmentPrintView?assignmentID=6248360 23/86
8/5/2018 Taller de Autoaprendizaje No 1a

In the special case that v r = vw , what would be Tim's margin of victory Δt(v r ?
= vw )

Hint 1. Think it through


If v r = vw , is the any difference between what Tim and Rick do?

ANSWER:

Δt(v r = v w ) = 0

Correct

± Average Velocity from a Position vs. Time Graph

Learning Goal:
To learn to read a graph of position versus time and to calculate average velocity.

In this problem you will determine the average velocity of a moving


object from the graph of its position x(t) as a function of time t. A
traveling object might move at different speeds and in different
directions during an interval of time, but if we ask at what constant
velocity the object would have to travel to achieve the same
displacement over the given time interval, that is what we call the
object's average velocity. We will use the notation v ave [t1 , t2 ] to
indicate average velocity over the time interval from t1 to t2 . For
instance, v ave [1, 3] is the average velocity over the time interval from
t = 1 to t = 3 .

Part A
Consulting the graph shown in the figure, find the object's average velocity over the time interval from 0 to 1 second.
Answer to the nearest integer.

Hint 1. Definition of average velocity


Average velocity is defined as the constant velocity at which an object would have to travel to achieve a given
displacement (difference between final and initial positions, which can be negative) over a given time interval, from
the initial time ti to the final time tf . The average velocity is therefore equal to the displacement divided by the given
time interval. In symbolic form, average velocity is given by

x(t f )−x(t i )
vave [ti , tf ] = .
t f −t i

ANSWER:

https://session.masteringphysics.com/myct/assignmentPrintView?assignmentID=6248360 24/86
8/5/2018 Taller de Autoaprendizaje No 1a

v ave [0, 1] = 0 m/s

Correct

Part B
Find the average velocity over the time interval from 1 to 3 seconds.
Express your answer in meters per second to the nearest integer.

Hint 1. Find the change in position


The final and initial positions can be read off the y axis of the graph. What is the displacement during the time interval
from 1 to 3 seconds?
Express your answer numerically, in meters
ANSWER:

xf − xi = 40 m

Correct

Hint 2. Definition of average velocity


Average velocity is defined as the constant velocity at which an object would have to travel to achieve a given
displacement (difference between final and initial positions, which can be negative) over a given time interval, from
the initial time ti to the final time tf . The average velocity is therefore equal to the displacement divided by the given
time interval. In symbolic form, average velocity is given by

x(t f )−x(t i )
vave [ti , tf ] = .
t f −t i

ANSWER:

v ave [1, 3] = 20 m/s

Correct
A note about instantaneous velocity. The instantaneous velocity at a certain moment in time is represented by the
slope of the graph at that moment. For straight-line graphs, the (instantaneous) velocity remains constant over the
interval, so the instantaneous velocity at any time during an interval is the same as the average velocity over that
interval. For instance, in this case, the instantaneous velocity at any time from 1 to 3 seconds is the same as the
average velocity of 20 m/s.

Part C
Now find v ave [0, 3].

Give your answer to three significant figures.

https://session.masteringphysics.com/myct/assignmentPrintView?assignmentID=6248360 25/86
8/5/2018 Taller de Autoaprendizaje No 1a

Hint 1. A note on the displacement


Since the object's position remains constant from time 0 to time 1, the object's displacement from 0 to 3 is the same
as in Part B. However, the time interval has changed.

ANSWER:

v ave [0, 3] = 13.3 m/s

Correct
vave [0,1]+vave [1,3]
Note that v ave [0, 3] is not equal to the simple arithmetic average of v ave [0, 1] and v ave [1, 3], i.e.,
2

, because they are averages for time intervals of different lengths.

Part D
Find the average velocity over the time interval from 3 to 6 seconds.

Express your answer to three significant figures.

Hint 1. Determine the displacement


What is the displacement?
Answer to the nearest integer.
ANSWER:

x(6.0) − x(3.0) = -40 m

Correct

Hint 2. Determine the time interval


What is the time interval?
Answer to two significant figures.
ANSWER:

tf − ti = 3.0 s

ANSWER:

v ave [3.0, 6.0] = -13.3 m/s

Correct

https://session.masteringphysics.com/myct/assignmentPrintView?assignmentID=6248360 26/86
8/5/2018 Taller de Autoaprendizaje No 1a

Part E
Finally, find the average velocity over the whole time interval shown in the graph.
Express your answer to three significant figures.

Hint 1. Determine the displacement


What is the displacement?
Answer to the nearest integer.

ANSWER:

x(6.0) − x(0.0) = 0 m

Correct

ANSWER:

v ave [0.0, 6.0] = 0 m/s

Correct
Note that though the average velocity is zero for this time interval, the instantaneous velocity (i.e., the slope of the
graph) has several different values (positive, negative, zero) during this time interval.

Note as well that since average velocity over a time interval is defined as the change in position (displacement) in
the given interval divided by the time, the object can travel a great distance (here 80 meters) and still have zero
average velocity, since it ended up exactly where it started. Therefore, zero average velocity does not necessarily
mean that the object was standing still the entire time!

What x vs. t Graphs Can Tell You

To describe the motion of a particle along a straight line, it is often convenient to draw a graph representing the position of the
particle at different times. This type of graph is usually referred to as an x vs. t graph. To draw such a graph, choose an axis
system in which time t is plotted on the horizontal axis and position x on the vertical axis. Then, indicate the values of x at
various times t. Mathematically, this corresponds to plotting the variable x as a function of t. An example of a graph of position
as a function of time for a particle traveling along a straight line is shown below. Note that an x vs. t graph like this does not
represent the path of the particle in space.

Now let's study the graph shown in the figure in more detail. Refer to this graph to answer Parts A, B, and C.

https://session.masteringphysics.com/myct/assignmentPrintView?assignmentID=6248360 27/86
8/5/2018 Taller de Autoaprendizaje No 1a

Part A
What is the overall displacement Δx of the particle?
Express your answer in meters.

Hint 1. Definition of displacement


The displacement Δx of the particle is given by the difference between the initial position x0 at t = 0.0 s and the
position x at t = 50.0 s. In symbols,

Δx = x − x0 .

Hint 2. How to read an x vs. t graph


Remember that in an x vs. t graph, time t is plotted on the horizontal axis and position x on the vertical axis. For
example, in the plot shown in the figure, x = 16.0 m at t = 10.0 s.

ANSWER:

Δx = 30 m

Correct
In this example, the magnitude of the displacement is also equal to the total distance traveled by the particle (30
m).

Part B
What is the average velocity v av of the particle over the time interval Δt = 50.0 s ?
Express your answer in meters per second.

Hint 1. Definition and graphical interpretation of average velocity


The average velocity v av of a particle that undergoes a displacement Δx along a straight line in a time interval Δt is
defined as

https://session.masteringphysics.com/myct/assignmentPrintView?assignmentID=6248360 28/86
8/5/2018 Taller de Autoaprendizaje No 1a
Δx
vav = .
Δt

In an x vs. t graph, then, the average velocity equals the slope of the line connecting the initial and final positions.

Hint 2. Slope of a line


The slope m of a line from point A, with coordinates (tA , xA ), to point B, with coordinates (tB , xB ), is equal to the
"rise" over the "run," or
x B −x A
m= .
t B −t A

ANSWER:

v av = 0.600 m/s

Correct
The average velocity of a particle between two positions is equal to the slope of the line connecting the two
corresponding points in an x vs. t graph.

Part C
What is the instantaneous velocity v of the particle at t = 10.0 s ?
Express your answer in meters per second.

Hint 1. Graphical interpretation of instantaneous velocity


The velocity of a particle at any given instant of time or at any point in its path is called instantaneous velocity. In an x
vs. t graph of the particle's motion, you can determine the instantaneous velocity of the particle at any point in the
curve. The instantaneous velocity at any point is equal to the slope of the line tangent to the curve at that point.

ANSWER:

v = 0.600 m/s

Correct
The instantaneous velocity of a particle at any point on its x vs. t graph is the slope of the line tangent to the curve
at that point. Since in the case at hand the curve is a straight line, the tangent line is the curve itself. Physically, this
means that the instantaneous velocity of the particle is constant over the entire time interval of motion. This is true
for any motion where distance increases linearly with time.

Another common graphical representation of motion along a straight line is the v vs. t graph, that is, the graph of (instantaneous)
velocity as a function of time. In this graph, time t is plotted on the horizontal axis and velocity v on the vertical axis. Note that by
definition, velocity and acceleration are vector quantities. In straight-line motion, however, these vectors have only one nonzero
component in the direction of motion. Thus, in this problem, we will call v the velocity and a the acceleration, even though they
are really the components of the velocity and acceleration vectors in the direction of motion.

Part D

https://session.masteringphysics.com/myct/assignmentPrintView?assignmentID=6248360 29/86
8/5/2018 Taller de Autoaprendizaje No 1a
Which of the graphs shown is the correct v vs. t plot for the motion described in the previous parts?

Hint 1. How to approach the problem


Recall your results found in the previous parts, namely the fact that the instantaneous velocity of the particle is
constant. Which graph represents a variable that always has the same constant value at any time?

ANSWER:

Graph A

Graph B

Graph C

Graph D

Correct
Whenever a particle moves with constant nonzero velocity, its x vs. t graph is a straight line with a nonzero slope,
and its v vs. t curve is a horizontal line.

Part E
Shown in the figure is the v vs. t curve selected in the previous part. What is the area A of the shaded region under the
curve?
Express your answer in meters.

https://session.masteringphysics.com/myct/assignmentPrintView?assignmentID=6248360 30/86
8/5/2018 Taller de Autoaprendizaje No 1a

Hint 1. How to approach the problem


The shaded region under the v vs. t curve is a rectangle whose horizontal and vertical sides lie on the t axis and the
v axis, respectively. Since the area of a rectangle is the product of its sides, in this case the area of the shaded region

is the product of a certain quantity expressed in seconds and another quantity expressed in meters per second. The
area itself, then, will be in meters.

ANSWER:

A = 30 m

Correct
Compare this result with what you found in Part A. As you can see, the area of the region under the v vs. t curve
equals the overall displacement of the particle. This is true for any velocity curve and any time interval: The area of
the region that extends over a time interval Δt under the v vs. t curve is always equal to the displacement over Δt
.

Given Positions, Find Velocity and Acceleration

Learning Goal:
To understand how to graph position, velocity, and acceleration of an object starting with a table of positions vs. time.
The table shows the x coordinate of a moving object. The position is tabulated at 1-s intervals. The x coordinate is indicated
below each time. You should make the simplification that the acceleration of the object is bounded and contains no spikes.

time (s) 0 1 2 3 4 5 6 7 8 9
x (m) 0 1 4 9 16 24 32 40 46 48

Part A

Which graph in best represents the function x(t), describing the object's position vs. time?

https://session.masteringphysics.com/myct/assignmentPrintView?assignmentID=6248360 31/86
8/5/2018 Taller de Autoaprendizaje No 1a

Hint 1. Meaning of a bounded and nonspiky acceleration


A bounded and nonspiky acceleration results in a smooth graph of x vs. t.

ANSWER:

Graph 1

Graph 2

Graph 3

Graph 4

Correct

Part B

Which of the following graphs in best represents the function v(t),


describing the object's velocity as a function of time?

Hint 1. Find the velocity toward the end of the motion


https://session.masteringphysics.com/myct/assignmentPrintView?assignmentID=6248360 32/86
8/5/2018 Taller de Autoaprendizaje No 1a
Velocity is the time derivative of displacement. Given this, the velocity toward the end of the motion is __________.
ANSWER:

positive and increasing

positive and decreasing

negative and increasing

negative and decreasing

Hint 2. What are the implications of zero velocity?


Two of the possible velocity vs. time graphs indicate zero velocity between t = 4 and t = 7 s . What would the
corresponding position vs. time graph look like in this region?

ANSWER:

a horizontal line

straight but sloping up to the right

straight but sloping down to the right

curved upward

curved downward

Hint 3. Specify the characteristics of the velocity function


The problem states that "the acceleration of the object is bounded and contains no spikes." This means that the
velocity ___________.
ANSWER:

has spikes

has no discontinuities

has no abrupt changes of slope

is constant

ANSWER:

https://session.masteringphysics.com/myct/assignmentPrintView?assignmentID=6248360 33/86
8/5/2018 Taller de Autoaprendizaje No 1a

Graph 1

Graph 2

Graph 3

Graph 4

Correct
In principle, you could also just compute and plot the average velocity. The expression for the average velocity is
x(t 2 )−x(t 1 )
vavg [t1 , t2 ] = .
t 2 −t 1

The notation v avg [t1 , t2 ] emphasizes that this is not an instantaneous velocity, but rather an average over an
interval. After you compute this, you must put a single point on the graph of velocity vs. time. The most accurate
place to plot the average velocity is at the middle of the time interval over which the average was computed.

Also, you could work back and find the position from the velocity graph. The position of an object is the integral of
its velocity. That is, the area under the graph of velocity vs. time from t = 0 up to time t must equal the position of
the object at time t. Check that the correct velocity vs. time graph gives you the correct position according to this
method.

Part C
Which of the following graphs in best represents the function a(t),
describing the acceleration of this object?

Hint 1. Find the acceleration toward the end of the motion


Acceleration is the time derivative of velocity. Toward the end of the motion the acceleration is __________.
ANSWER:

zero

positive

negative

https://session.masteringphysics.com/myct/assignmentPrintView?assignmentID=6248360 34/86
8/5/2018 Taller de Autoaprendizaje No 1a

Hint 2. Calculate the acceleration in the region of constant velocity


What is the acceleration a over the interval during which the object travels at constant speed?
Answer numerically in meters per second squared.
ANSWER:

a = 0 m/s
2

Hint 3. Find the initial acceleration


Acceleration is the time derivative of velocity. Initially the acceleration is _________.
ANSWER:

zero

positive

negative

ANSWER:

Graph 1

Graph 2

Graph 3

Graph 4

Correct
In one dimension, a linear increase or decrease in the velocity of an object over a given time interval implies
constant acceleration over that particular time interval. You can find the magnitude of the acceleration using the
formula for average acceleration over a time interval:
v(t 2 )−v(t 1 )
aavg [t1 , t2 ] = .
t 2 −t 1

When the acceleration is constant over an extended interval, you can choose any value of t1 and t2 within the
interval to compute the average.

Kinematic Vocabulary

One of the difficulties in studying mechanics is that many common words are used with highly specific technical meanings,
among them velocity, acceleration, position, speed, and displacement. The series of questions in this problem is designed to get
you to try to think of these quantities like a physicist.

Answer the questions in this problem using words from the following list:

A. position
B. direction
https://session.masteringphysics.com/myct/assignmentPrintView?assignmentID=6248360 35/86
8/5/2018 Taller de Autoaprendizaje No 1a
C. displacement
D. coordinates
E. velocity
F. acceleration
G. distance
H. magnitude
I. vector
J. scalar
K. components

Part A
Velocity differs from speed in that velocity indicates a particle's __________ of motion.

Enter the letter from the list given in the problem introduction that best completes the sentence.
ANSWER:

Correct

Part B
Unlike speed, velocity is a __________ quantity.
Enter the letter from the list given in the problem introduction that best completes the sentence.
ANSWER:

Correct

Part C
A vector has, by definition, both __________ and direction.
Enter the letter from the list given in the problem introduction that best completes the sentence.
ANSWER:

Correct

Part D
Once you have selected a coordinate system, you can express a two-dimensional vector using a pair of quantities known
collectively as __________.

Enter the letter from the list given in the problem introduction that best completes the sentence.

https://session.masteringphysics.com/myct/assignmentPrintView?assignmentID=6248360 36/86
8/5/2018 Taller de Autoaprendizaje No 1a
ANSWER:

Correct

Part E
Speed differs from velocity in the same way that __________ differs from displacement.
Enter the letter from the list given in the problem introduction that best completes the sentence.

Hint 1. Definition of displacement


Displacement is the vector that indicates the difference of two positions (e.g., the final position from the initial
position). Being a vector, it is independent of the coordinate system used to describe it (although its vector
components depend on the coordinate system).

ANSWER:

Correct

Part F
Consider a physical situation in which a particle moves from point A to point B. This process is described from two
coordinate systems that are identical except that they have different origins.

The __________ of the particle at point A differ(s) as expressed in one coordinate system compared to the other, but the
__________ from A to B is/are the same as expressed in both coordinate systems.
Type the letters from the list given in the problem introduction that best complete the sentence. Separate the letters
with commas. There is more than one correct answer, but you should only enter one pair of comma-separated
letters. For example, if the words "vector" and "scalar" fit best in the blanks, enter I,J.
ANSWER:

A,C

Correct
The coordinates of a point will depend on the coordinate system that is chosen, but there are several other
quantities that are independent of the choice of origin for a coordinate system: in particular, distance, displacement,
direction, and velocity. In working physics problems, unless you are interested in the position of an object or event
relative to a specific origin, you can usually choose the coordinate system origin to be wherever is most convenient
or intuitive.

Note that the vector indicating a displacement from A to B is usually represented as r BA


⃗  ⃗  − r ⃗ 
= rB A
.

Part G

https://session.masteringphysics.com/myct/assignmentPrintView?assignmentID=6248360 37/86
8/5/2018 Taller de Autoaprendizaje No 1a
Identify the following physical quantities as scalars or vectors.
ANSWER:

Reset Help

Scalar quantity Vector quantity


velocity average velocity

distance speed position displacement

acceleration

Correct

What Velocity vs. Time Graphs Can Tell You

A common graphical representation of motion along a straight line is the v vs. t graph, that is, the graph of (instantaneous)
velocity as a function of time. In this graph, time t is plotted on the horizontal axis and velocity v on the vertical axis. Note that by
definition, velocity and acceleration are vector quantities. In straight-line motion, however, these vectors have only a single
nonzero component in the direction of motion. Thus, in this problem, we will call v the velocity and a the acceleration, even
though they are really the components of the velocity and acceleration vectors in the direction of motion, respectively.

Here is a plot of velocity versus time for a particle that travels along a straight line with a varying velocity. Refer to this plot to
answer the following questions.

Part A
What is the initial velocity of the particle, v 0 ?
https://session.masteringphysics.com/myct/assignmentPrintView?assignmentID=6248360 38/86
8/5/2018 Taller de Autoaprendizaje No 1a
Express your answer in meters per second.

Hint 1. Initial velocity


The initial velocity is the velocity at t = 0 s .

Hint 2. How to read a v vs. t graph


Recall that in a graph of velocity versus time, time is plotted on the horizontal axis and velocity on the vertical axis.
For example, in the plot shown in the figure, v = 2.00 m/s at t = 30.0 s.

ANSWER:

v0 = 0.5 m/s

Correct

Part B
What is the total distance Δx traveled by the particle?
Express your answer in meters.

Hint 1. How to approach the problem


Recall that the area of the region that extends over a time interval Δt under the v vs. t curve is always equal to the
distance traveled in Δt. Thus, to calculate the total distance, you need to find the area of the entire region under the
v vs. t curve. In the case at hand, the entire region under the v vs. t curve is not an elementary geometrical figure, but
rather a combination of triangles and rectangles.

Hint 2. Find the distance traveled in the first 20.0 seconds


What is the distance Δx1 traveled in the first 20 seconds of motion, between t = 0.0 s and t = 20.0 s ?
Express your answer in meters.

Hint 1. Area of the region under the v vs. t curve


The region under the v vs. t curve between t = 0.0 s and t = 20.0 s can be divided into a rectangle of
dimensions 20.0 s by 0.50 m/s, and a triangle of base 20.0 s and height 1.50 m/s, as shown in the figure.

https://session.masteringphysics.com/myct/assignmentPrintView?assignmentID=6248360 39/86
8/5/2018 Taller de Autoaprendizaje No 1a

ANSWER:

Δx1 = 25 m

Hint 3. Find the distance traveled in the second 20.0 seconds


What is the distance Δx2 traveled in the second 20 seconds of motion, from t = 20.0 s to t = 40.0 s ?
Express your answer in meters.

Hint 1. Area of the region under the v vs. t curve


The region under the v vs. t curve between t = 20.0 s and t = 40.0 s is a rectangle of dimensions 20.0 s by
2.00 m/s, as shown in the figure.

ANSWER:

Δx2 = 40 m

Hint 4. Find the distance traveled in the last 10.0 seconds

https://session.masteringphysics.com/myct/assignmentPrintView?assignmentID=6248360 40/86
8/5/2018 Taller de Autoaprendizaje No 1a
What is the distance Δx3 traveled in the last 10 seconds of motion, from t = 40.0 s to t = 50.0 s ?
Express your answer in meters.

Hint 1. Area of the region under the v vs. t curve


The region under the v vs. t curve between t = 40.0 s and t = 50.0 s is a triangle of base 10.0 s and height
2.00 m/s, as shown in the figure.

ANSWER:

Δx3 = 10 m

ANSWER:

Δx = 75 m

Correct

Part C
What is the average acceleration aav of the particle over the first 20.0 seconds?
Express your answer in meters per second per second.

Hint 1. Definition and graphical interpretation of average acceleration


The average acceleration aav of a particle that travels along a straight line in a time interval Δt is the ratio of the
change in velocity Δv experienced by the particle to the time interval Δt, or

Δv
aav = .
Δt
In a v vs. t graph, then, the average acceleration equals the slope of the line connecting the two points representing
the initial and final velocities.

Hint 2. Slope of a line


https://session.masteringphysics.com/myct/assignmentPrintView?assignmentID=6248360 41/86
8/5/2018 Taller de Autoaprendizaje No 1a

The slope m of a line from point A, of coordinates (xA , yA ), to point B, of coordinates (xB , yB ), is equal to the "rise"
over the "run," or
y −y
m=
B

x B −x A
A
.

ANSWER:

aav = 0.075 m/s


2

Correct
The average acceleration of a particle between two instants of time is the slope of the line connecting the two
corresponding points in a v vs. t graph.

Part D
What is the instantaneous acceleration a of the particle at t = 45.0 s ?

Hint 1. Graphical interpretation of instantaneous acceleration


The acceleration of a particle at any given instant of time or at any point in its path is called the instantaneous
acceleration. If the v vs. t graph of the particle's motion is known, you can directly determine the instantaneous
acceleration at any point on the curve. The instantaneous acceleration at any point is equal to the slope of the line
tangent to the curve at that point.

Hint 2. Slope of a line


The slope m of a line from point A, of coordinates (xA , yA ), to point B, of coordinates (xB , yB ), is equal to the "rise"
over the "run," or
y B −y A
m=
x B −x A
.

ANSWER:

1 m/s2

0.20 m/s2

a = -0.20 m/s2

0.022 m/s2

-0.022 m/s2

https://session.masteringphysics.com/myct/assignmentPrintView?assignmentID=6248360 42/86
8/5/2018 Taller de Autoaprendizaje No 1a

Correct
The instantaneous acceleration of a particle at any point on a v vs. t graph is the slope of the line tangent to the
curve at that point. Since in the last 10 seconds of motion, between t = 40.0 s and t = 50.0 s, the curve is a
straight line, the tangent line is the curve itself. Physically, this means that the instantaneous acceleration of the
particle is constant over that time interval. This is true for any motion where velocity increases linearly with time. In
the case at hand, can you think of another time interval in which the acceleration of the particle is constant?

Now that you have reviewed how to plot variables as a function of time, you can use the same technique and draw an
acceleration vs. time graph, that is, the graph of (instantaneous) acceleration as a function of time. As usual in these types of
graphs, time t is plotted on the horizontal axis, while the vertical axis is used to indicate acceleration a.

Part E
Which of the graphs shown below is the correct acceleration vs. time plot for the motion described in the previous parts?

Hint 1. How to approach the problem


Recall that whenever velocity increases linearly with time, acceleration is constant. In the example here, the particle's
velocity increases linearly with time in the first 20.0 s of motion. In the second 20.0 s , the particle's velocity is
constant, and then it decreases linearly with time in the last 10 s. This means that the particle's acceleration is
constant over each time interval, but its value is different in each interval.

Hint 2. Find the acceleration in the first 20 s


What is a1 , the particle's acceleration in the first 20 s of motion, between t = 0.0 s and t = 20.0 s ?
Express your answer in meters per second per second.

Hint 1. Constant acceleration

https://session.masteringphysics.com/myct/assignmentPrintView?assignmentID=6248360 43/86
8/5/2018 Taller de Autoaprendizaje No 1a
Since we have already determined that in the first 20 s of motion the particle's acceleration is constant, its
constant value will be equal to the average acceleration that you calculated in Part C.

ANSWER:

a1 = 0.075 m/s
2

Hint 3. Find the acceleration in the second 20 s


What is a2 , the particle's acceleration in the second 20 s of motion, between t = 20.0 s and t = 40.0 s ?
Express your answer in meters per second per second.

Hint 1. Constant velocity


In the second 20 s of motion, the particle's velocity remains unchanged. This means that in this time interval,
the particle does not accelerate.

ANSWER:

a2 = 0 m/s
2

Hint 4. Find the acceleration in the last 10 s


What is a3 , the particle's acceleration in the last 10 s of motion, between t = 40.0 s and t = 50.0 s ?
Express your answer in meters per second per second.

Hint 1. Constant acceleration


Since we have already determined that in the last 10 s of motion the particle's acceleration is constant, its
constant value will be equal to the instantaneous acceleration that you calculated in Part D.

ANSWER:

a3 = -0.20 m/s
2

ANSWER:

Graph A

Graph B

Graph C

Graph D

https://session.masteringphysics.com/myct/assignmentPrintView?assignmentID=6248360 44/86
8/5/2018 Taller de Autoaprendizaje No 1a

Correct
In conclusion, graphs of velocity as a function of time are a useful representation of straight-line motion. If read
correctly, they can provide you with all the information you need to study the motion.

Velocity and Acceleration of a Power Ball

Learning Goal:
To understand the distinction between velocity and acceleration with the use of motion diagrams.
In common usage, velocity and acceleration both can imply having considerable speed. In physics, they are sharply defined
concepts that are not at all synonymous. Distinguishing clearly between them is a prerequisite to understanding motion.
Moreover, an easy way to study motion is to draw a motion diagram, in which the position of the object in motion is sketched at
several equally spaced instants of time, and these sketches (or snapshots) are combined into one single picture.

In this problem, we make use of these concepts to study the motion of a power ball. This discussion assumes that we have
already agreed on a coordinate system from which to measure the position r (t) ⃗  (also called the position vector) of objects as a
function of time. Let v (t) and a(t) be velocity and acceleration, respectively.
 
⃗  

Consider the motion of a power ball that is dropped on the floor and bounces back. In the following questions, you will describe
its motion at various points in its fall in terms of its velocity and acceleration.

Part A
You drop a power ball on the floor. The motion diagram of the ball is sketched in the figure . Indicate whether the magnitude
of the velocity of the ball is increasing, decreasing, or not
changing.

Hint 1. Velocity and displacement vectors


By definition, the velocity is the ratio of the distance traveled to the interval of time taken. If you interpret the vector
displacement Δr ⃗ as the distance traveled by the ball, the length of v ⃗ is directly proportional to the length of Δr .⃗ 
Since the length of displacement vectors is increasing, so is the length of velocity vectors.

ANSWER:

https://session.masteringphysics.com/myct/assignmentPrintView?assignmentID=6248360 45/86
8/5/2018 Taller de Autoaprendizaje No 1a

increasing

decreasing

not changing

Correct
While the ball is in free fall, the magnitude of its velocity is increasing, so the ball is accelerating.

Part B

Since the length of v ⃗ is directly proportional to the length of Δr ,⃗  the vector connecting each dot to the next could represent
velocity vectors as well as displacement vectors, as shown in the figure here . Indicate whether the velocity and acceleration
of the ball are, respectively, positive (upward), negative, or zero.
Use P, N, and Z for positive (upward), negative, and zero,
respectively. Separate the letters for velocity and acceleration
with a comma.

Hint 1. Acceleration vector


The acceleration is defined as the ratio of the change in velocity to the interval of time, and its direction is given by the
quantity Δv ⃗ = v (⃗  t2 ) − v (⃗  t1 ), which represents the change in velocity that occurs in the interval of time
Δt = t 2 − t 1 .

ANSWER:

N,N

Correct

Part C
Now, consider the motion of the power ball once it bounces upward. Its motion diagram is shown in the figure here . Indicate
whether the magnitude of the velocity of the ball is increasing, decreasing, or not changing.

https://session.masteringphysics.com/myct/assignmentPrintView?assignmentID=6248360 46/86
8/5/2018 Taller de Autoaprendizaje No 1a

Hint 1. Velocity and displacement vectors


By definition, the velocity is the ratio of the distance traveled to the interval of time taken. If you interpret the vector
displacement Δr ⃗ as the distance traveled by the ball, the length of v ⃗ is directly proportional to the length of Δr .⃗ 
Since the length of displacement vectors is decreasing, so is the length of velocity vectors.

ANSWER:

increasing

decreasing

not changing

Correct
Since the magnitude of the velocity of the ball is decreasing, the ball must be accelerating (specifically, slowing
down).

Part D
The next figure shows the velocity vectors corresponding to the
upward motion of the power ball. Indicate whether its velocity and
acceleration, respectively, are positive (upward), negative, or zero.

Use P, N, and Z for positive (upward), negative, and zero,


respectively. Separate the letters for velocity and acceleration
with a comma.

https://session.masteringphysics.com/myct/assignmentPrintView?assignmentID=6248360 47/86
8/5/2018 Taller de Autoaprendizaje No 1a

Hint 1. Acceleration vector


The acceleration is defined as the ratio of the change in velocity to the interval of time, and its direction is given by the
quantity Δv ⃗ = v (⃗  t2 ) − v (⃗  t1 ), which represents the change in velocity that occurs in the interval of time
Δt = t 2 − t 1 .

ANSWER:

P,N

Correct

Part E
The power ball has now reached its highest point above the ground and starts to descend again. The motion diagram
representing the velocity vectors is the same as that after the initial release, as shown in the figure of Part B. Indicate
whether the velocity and acceleration of the ball at its highest point are positive (upward), negative, or zero.
Use P, N, and Z for positive (upward), negative, and zero, respectively. Separate the letters for velocity and
acceleration with a comma.

Hint 1. Velocity as a continuous function of time


In Part D you found that the velocity of the ball is positive during the upward motion. Once the ball starts its descent,
its velocity is negative, as you found in Part B. Since velocity changes continuously in time, it has to be zero at some
point along the path of the ball.

Hint 2. Acceleration as a continuous function of time


In Part D, you found that the acceleration of the ball is negative and constant during the upward motion, as well as
once the ball has started its descent, which you found in Part B. Since acceleration is a continuous function of time, it
has to be negative at the highest point along the path as well.

ANSWER:

Z,N

Correct
These examples should show you that the velocity and acceleration can have opposite or similar signs or that one
of them can be zero while the other has either sign. Try hard to think carefully about them as distinct physical
quantities when working with kinematics.

One-Dimensional Kinematics with Constant Acceleration

Learning Goal:

To understand the meaning of the variables that appear in the equations for one-dimensional kinematics with constant
acceleration.

https://session.masteringphysics.com/myct/assignmentPrintView?assignmentID=6248360 48/86
8/5/2018 Taller de Autoaprendizaje No 1a
Motion with a constant, nonzero acceleration is not uncommon in the world around us. Falling (or thrown) objects and cars
starting and stopping approximate this type of motion. It is also the type of motion most frequently involved in introductory
kinematics problems.

The kinematic equations for such motion can be written as

1 2
x(t) = x i + vi t + at ,
2

v(t) = v i + at ,

where the symbols are defined as follows:

x(t) is the position of the particle;


xi is the initial position of the particle;
v(t) is the velocity of the particle;

v i is the initial velocity of the particle;

a is the acceleration of the particle.

In anwering the following questions, assume that the acceleration is constant and nonzero: a ≠ 0 .

Part A
The quantity represented by x is a function of time (i.e., is not constant).
ANSWER:

true

false

Correct

Part B
The quantity represented by xi is a function of time (i.e., is not constant).
ANSWER:

true

false

Correct
Recall that xi represents an initial value, not a variable. It refers to the position of an object at some initial moment.

Part C
The quantity represented by v i is a function of time (i.e., is not constant).

ANSWER:

https://session.masteringphysics.com/myct/assignmentPrintView?assignmentID=6248360 49/86
8/5/2018 Taller de Autoaprendizaje No 1a

true

false

Correct

Part D
The quantity represented by v is a function of time (i.e., is not constant).
ANSWER:

true

false

Correct
The velocity v always varies with time when the linear acceleration is nonzero.

Part E
Which of the given equations is not an explicit function of t and is therefore useful when you don't know or don't need the
time?

ANSWER:

1 2
x = x i + vi t + at
2

v = v i + at

2 2
v = v + 2a(x − xi )
i

Correct

Part F
A particle moves with constant acceleration a. The expression v i + at represents the particle's velocity at what instant in
time?

ANSWER:

https://session.masteringphysics.com/myct/assignmentPrintView?assignmentID=6248360 50/86
8/5/2018 Taller de Autoaprendizaje No 1a

only at time t = 0

only at the "initial" time

when a time t has passed since the particle's velocity was v i

Correct

More generally, the equations of motion can be written as


1 2
x(t) = x i + vi Δt + a (Δt)
2
and

v(t) = v i + a Δt.

Here Δt is the time that has elapsed since the beginning of the particle's motion, that is, Δt = t − ti , where t is the current time
and ti is the time at which we start measuring the particle's motion. The terms xi and v i are, respectively, the position and
velocity at t = ti . As you can now see, the equations given at the beginning of this problem correspond to the case ti = 0,
which is a convenient choice if there is only one particle of interest.

To illustrate the use of these more general equations, consider the motion of two particles, A and B. The position of particle A
depends on time as xA (t) = xi + v i t + (1/2)at2 . That is, particle A starts moving at time t = tiA = 0 with velocity v iA = v i ,
from xiA = xi . At time t = t1 , particle B has twice the acceleration, half the velocity, and the same position that particle A had at
time t = 0 .

Part G
What is the equation describing the position of particle B?

Hint 1. How to approach the problem


The general equation for the distance traveled by particle B is
1 2
x B (t) = x iB + viB Δt + aB (Δt) ,
2
or

1 2
x B (t) = x B (t = t1 ) + vB (t = t1 )(t − t1 ) + aB (t − t1 ) ,
2
since Δt = t − t1 is a good choice for B. From the information given, deduce the correct values of the constants
that go into the equation for xB (t) given here, in terms of A's constants of motion.

ANSWER:

https://session.masteringphysics.com/myct/assignmentPrintView?assignmentID=6248360 51/86
8/5/2018 Taller de Autoaprendizaje No 1a

1 2
x B (t) = x i + 2vi t + at
4

2
xB (t) = xi + 0.5v i t + at

1 2
x B (t) = x i + 2vi (t + t1 ) + a(t + t1 )
4

2
xB (t) = xi + 0.5v i (t + t 1 ) + a(t + t 1 )

1 2
x B (t) = x i + 2vi (t − t1 ) + a(t − t1 )
4

2
xB (t) = xi + 0.5v i (t − t 1 ) + a(t − t 1 )

Correct

Part H
At what time does the velocity of particle B equal that of particle A?

Hint 1. Velocity of particle A


Type an expression for particle A's velocity as a function of time.
Express your answer in terms of t and some or all of the variables xi , v i , and a.

Hint 1. How to approach this part


Look at the general expression for v(t) given in the problem introduction.

ANSWER:

v A (t) = v i + at

Hint 2. Velocity of particle B


Type an expression for particle B's velocity as a function of time.

Express your answer in terms of t and some or all of the variables t1 , xi , v i , and a.

Hint 1. How to approach this part


The general expression for v B (t) is

v B (t) = v B (t = t 1 ) + aB (t − t 1 ) .

From the information given, deduce the correct values of the constants that go into this equation in terms of
particle A's constants of motion.

ANSWER:

https://session.masteringphysics.com/myct/assignmentPrintView?assignmentID=6248360 52/86
8/5/2018 Taller de Autoaprendizaje No 1a

v B (t) = 0.5v i + 2a(t − t 1 )

ANSWER:

vi
t = t1 +
4a

vi
t = 2t1 +
2a

vi
t = 3t1 +
2a

The two particles never have the same velocity.

Correct

PSS 2.1 Motion with Constant Acceleration

Learning Goal:

To practice Problem-Solving Strategy 2.1 Motion with Constant Acceleration.


Cheetahs, the fastest of the great cats, can reach 50.0 miles/hour in 2.22 s starting from rest. Assuming that they have
constant acceleration throughout that time, find their acceleration in meters per second squared.

Problem-Solving Strategy: Motion with constant acceleration

IDENTIFY the relevant concepts:


In most straight-line motion problems, you can use the constant-acceleration equations. Occasionally, however, you will
encounter a situation in which the acceleration isn’t constant. In such a case, you’ll need a different approach.

SET UP the problem using the following steps:

1. First, decide where the origin of coordinates is and which axis direction is positive. It is often easiest to place the
particle at the origin at time t = 0 ; then x0 = 0. It helps to make a motion diagram showing the coordinates and
some later positions of the particle.
2. Keep in mind that your choice of the positive axis direction automatically determines the positive direction for x
velocity and x acceleration. If x is positive to the right of the origin, then v x and ax are also positive toward the
right.
3. Restate the problem in words, and then translate it into symbols and equations.
4. Make a list of known and unknown quantities such as x, x0 , v x , v 0x , ax , and t. Write down the values of the
known quantities, and decide which of the unknowns are the target variables. Look for implicit information.

EXECUTE the solution as follows:


Choose an equation from the following list

v x = v 0x + ax t

1 2
x = x 0 + v0x t + ax t
2

2 2
vx = v + 2ax (x − x0 )
0x

v0x +vx
x − x0 = ( )t
2

https://session.masteringphysics.com/myct/assignmentPrintView?assignmentID=6248360 53/86
8/5/2018 Taller de Autoaprendizaje No 1a
that contains only one of the target variables. Solve this equation for the target variable, using symbols only. Then, substitute the
known values and compute the value of the target variable. Sometimes you will have to solve two simultaneous equations for two
unknown quantities.

EVALUATE your answer:


Take a hard look at your results to see whether they make sense. Are they within the general range of values you expected?

IDENTIFY the relevant concepts

This problem involves the motion of an object, the cheetah, whose acceleration is assumed constant. Thus, the equations given
in this strategy apply.

SET UP the problem using the following steps

Part A
Which of the following sketches and choice of coordinate axis best describe the physical situation presented in this problem?

ANSWER:

Correct

Part B
The next step is to translate the problem statement from words into symbols. Which of the following is an appropriate
restatement of the problem, "Cheetahs, the fastest of the great cats, can reach 50.0 miles/hour in 2.22 s starting from rest.
Assuming that they have constant acceleration throughout that time, find their acceleration in meters per second squared."

Hint 1. Find the initial velocity using implicit information


The problem states that the cheetah starts running from rest. What is the initial velocity v 0x of the cheetah?
https://session.masteringphysics.com/myct/assignmentPrintView?assignmentID=6248360 54/86
8/5/2018 Taller de Autoaprendizaje No 1a
Enter your answer in meters per second.

ANSWER:

v 0x = 0 m/s

Hint 2. The condition for the equations of motion presented in this problem
The equations presented in the strategy above only apply to situations involving motion under constant acceleration.

ANSWER:

Cheetahs can reach v 0x = 50.0 miles/hour in t = 2.22 s starting from v x = 0 . What is ax ?

Cheetahs can reach v x = 50.0 miles/hour in t = 2.22 s starting from v 0x . What is ax ?


= 0

Cheetahs can reach v x = 50.0 miles/hour in t = 2.22 s starting from v 0x . Assuming a


= 0 = constant , what
is ax ?

Cheetahs can reach v 0x = 50.0 miles/hour in t = 2.22 s starting from v x = 0 . Assuming a = constant , what
is ax ?

Correct
Now you compile a list of known and unknown quantities. You can organize this information in a table as shown
below.
Known Unknown
x0 = 0 m x

v 0x = 0 m/s ax

vx = 50.0 miles/hour _

t = 2.22 s _

Keep in mind that your target variable is ax .

EXECUTE the solution as follows

Part C
Finally, you are ready to answer the main question. Cheetahs, the fastest of the great cats, can reach 50.0 miles/hour in
2.22 s starting from rest. Assuming that they have constant acceleration throughout that time, find their acceleration in
meters per second squared.
Enter your answer in meters per second squared to three significant figures.

Hint 1. Identify what equation to use


Which of the following equations would be the best to use when solving for ax ?

ANSWER:

https://session.masteringphysics.com/myct/assignmentPrintView?assignmentID=6248360 55/86
8/5/2018 Taller de Autoaprendizaje No 1a

1 2
x = x 0 + v0x t + ax t
2

v x = v 0x + ax t

v0x +vx
x − x0 = ( )t
2

2 2
vx = v + 2ax (x − x0 )
0x

Correct
Now, solve for ax . Before you substitute the known values, be sure to convert all quantities to SI units.

Hint 2. Convert to SI units


How many meters per second are equivalent to 50.0 miles/hour ?
Enter your answer in meters per second to three significant figures.

Hint 1. The conversion factor from miles to meters


To convert miles to meters, use
1 mile = 1609 m .

ANSWER:

50.0 miles/hour = 22.3 m/s

Correct

ANSWER:

ax = 10.1 m/s
2

Correct

EVALUATE your answer

Part D
Imagine you looked up the accelerations of the following objects: snails, humans, Thomson's gazelles, the space shuttle,
Formula One race cars, and F-16 fighter jets. Which of the following statements about the acceleration of a cheetah would
you expect to be true?

ANSWER:

https://session.masteringphysics.com/myct/assignmentPrintView?assignmentID=6248360 56/86
8/5/2018 Taller de Autoaprendizaje No 1a

The acceleration of a cheetah is greater than the acceleration of a snail but less than the acceleration of a
human.

The acceleration of a cheetah is greater than the acceleration of a Formula One race car but less than the
acceleration of an F-16 fighter jet.

The acceleration of a cheetah is greater than the acceleration of a Thomson's gazelle but less than the
acceleration of the space shuttle during liftoff.

Correct

The acceleration of the space shuttle on takeoff is 29.4 m/s2 . Thomson's gazelles can accelerate at approximately
half the rate of a cheetah, which is why they often become tasty snacks for the fast cats.

If you had solved for the acceleration of a cheetah and calculated a number greater than 29.4 m/s2 or smaller
than 30 cm/s2 (the acceleration of a snail), you most likely made an error and would want to review your work.

Motion of Two Rockets

Learning Goal:

To learn to use images of an object in motion to determine position, velocity, and acceleration.
Two toy rockets are traveling in the same direction (taken to be the x axis). A diagram is shown of a time-exposure image where
a stroboscope has illuminated the rockets at the uniform time intervals indicated.

Part A
At what time(s) do the rockets have the same velocity?

Hint 1. How to determine the velocity


The diagram shows position, not velocity. You can't find instantaneous velocity from this diagram, but you can
determine the average velocity between two times t1 and t2 :

x(t 2 )−x(t 1 )
vavg [t1 , t2 ] = .
t 2 −t 1

https://session.masteringphysics.com/myct/assignmentPrintView?assignmentID=6248360 57/86
8/5/2018 Taller de Autoaprendizaje No 1a
Note that no position values are given in the diagram; you will need to estimate these based on the distance between
successive positions of the rockets.

ANSWER:

at time t = 1 only

at time t = 4 only

at times t = 1 and t = 4

at some instant in time between t = 1 and t = 4

at no time shown in the figure

Correct

Part B
At what time(s) do the rockets have the same x position?

ANSWER:

at time t = 1 only

at time t = 4 only

at times t = 1 and t = 4

at some instant in time between t = 1 and t = 4

at no time shown in the figure

Correct

Part C
At what time(s) do the two rockets have the same acceleration?

Hint 1. How to determine the acceleration


The velocity is related to the spacing between images in a stroboscopic diagram. Since acceleration is the rate at
which velocity changes, the acceleration is related to the how much this spacing changes from one interval to the
next.

ANSWER:

https://session.masteringphysics.com/myct/assignmentPrintView?assignmentID=6248360 58/86
8/5/2018 Taller de Autoaprendizaje No 1a

at time t = 1 only

at time t = 4 only

at times t = 1 and t = 4

at some instant in time between t = 1 and t = 4

at no time shown in the figure

Correct

Part D
The motion of the rocket labeled A is an example of motion with uniform (i.e., constant) __________.
ANSWER:

and nonzero acceleration

velocity

position

time

Correct

Part E
The motion of the rocket labeled B is an example of motion with uniform (i.e., constant) __________.
ANSWER:

and nonzero acceleration

velocity

position

time

Correct

Part F
At what time(s) is rocket A ahead of rocket B?

Hint 1. Use the diagram


https://session.masteringphysics.com/myct/assignmentPrintView?assignmentID=6248360 59/86
8/5/2018 Taller de Autoaprendizaje No 1a
You can answer this question by looking at the diagram and identifying the time(s) when rocket A is to the right of
rocket B.

ANSWER:

before t = 1 only

after t = 4 only

before t = 1 and after t = 4

between t = 1 and t = 4

at no time(s) shown in the figure

Correct

± Rocket Height

A rocket, initially at rest on the ground, accelerates straight upward from rest with constant acceleration 53.9 m/s2 . The
acceleration period lasts for time 10.0 s until the fuel is exhausted. After that, the rocket is in free fall.

Part A
Find the maximum height ymax reached by the rocket. Ignore air resistance and assume a constant acceleration due to
gravity equal to 9.80 m/s2 .

Write your answer numerically in units of meters.

Hint 1. How to approach the problem


Divide the upward motion into two parts: first the fueled motion, and then the motion under the influence of gravity
alone. Find the height reached over the course of the fueled motion, and then calculate the additional height achieved
during the second part of the motion. Putting these two distances together will give you the maximum height reached
by the rocket.

Hint 2. Find the height reached during the fueled part of the motion
Find the height yf uel above the ground at which the rocket exhausts its fuel.

Answer numerically in units of meters.

Hint 1. Knowns and unknowns


At the instant that the rocket takes off, take time t0 = 0 and the initial position y0 = 0. Let the final values of
the variables correspond to those at which the rocket runs out of fuel. Clearly, the final height yf uel for the
fueled portion of the flight and the associated final velocity v f uel are not given.

Let us denote other quantities as follows: tf uel is the time that the rocket travels before it runs out of fuel; v 0,f uel
is the rocket's initial velocity; and af uel is the rocket's net acceleration during the fueled portion of its flight.
Which of the these quantities are known?
Check all that apply.

https://session.masteringphysics.com/myct/assignmentPrintView?assignmentID=6248360 60/86
8/5/2018 Taller de Autoaprendizaje No 1a

Hint 1. What is the initial velocity?


What is the initial velocity v 0,f uel for the fueled part of the motion?

Give your answer numerically.

ANSWER:

v 0,f uel = 0 m/s

ANSWER:

t f uel

v 0,f uel

af uel

Hint 2. Determine which kinematic equation to use


Choose the kinematic equation that makes the solution straighforward, that is, the one that contains the
variable you are solving for and in which all of the other quantities are known.
ANSWER:

v = v 0 + at

1 2
y= y + v0 t + at
0 2

2 2
v = v + 2a(y − y 0 )
0

v0 +v
y− y = ( )t
0 2

ANSWER:

y
f uel
= 2700 m

Hint 3. Find the initial velocity, the final velocity, and the acceleration for the "free-fall" part of the motion
What are v 0,grav , v f ,grav , and agrav for the second part of the motion?

Write your answer numerically in the order v 0,grav , v f ,grav , agrav , separated by commas as shown, in SI units.

Hint 1. What is the initial velocity?


When the rocket runs out of fuel, its acceleration changes abruptly, but its velocity changes continuously.
Therefore, the rocket's initial velocity v 0,grav for the second part of the flight is just its velocity at the moment
the engine runs out of fuel. What, then, is v 0,grav ?

https://session.masteringphysics.com/myct/assignmentPrintView?assignmentID=6248360 61/86
8/5/2018 Taller de Autoaprendizaje No 1a
Give your answer numerically.

Hint 1. Find the velocity when the engine runs out of fuel
For the fueled part of the motion, you know that the initial velocity is given by v 0,f uel = 0, the
acceleration by af uel = 53.9 m/s2 , and the time of fueled flight by tf uel = 10.0 s . You also
determined the height yf uel in Part A.2.
Choose a kinematic equation that you could use to find v f ,f uel , the velocity at the end of the fueled
motion.

1. v = v 0 + at
2. y = y0 + v 0 t + (1/2)at2
3. v 2 = v 20 + 2a(y − y0 )
v0 +v
4. y − y0 = ( )t
2

Choose one letter corresponding to the equation you have chosen (even though there is more
than one correct answer).

ANSWER:

ANSWER:

v 0,grav = 539 m/s

Hint 2. What is the acceleration?


What value should you use for the acceleration agrav ? Keep in mind that the direction is important, since the
acceleration due to gravity is slowing down the rocket as it continues its ascent.

Give your answer numerically.


ANSWER:

agrav = -9.80 m/s


2

Hint 3. What is the final velocity?


What is the velocity v f ,grav of the rocket when it reaches its maximum height? Note that the rocket has just
ended its ascent and is about to begin its descent. What is its velocity at this instant?
Give your answer numerically.
ANSWER:

https://session.masteringphysics.com/myct/assignmentPrintView?assignmentID=6248360 62/86
8/5/2018 Taller de Autoaprendizaje No 1a

v f ,grav = 0 m/s

ANSWER:

v 0,grav , v f ,grav , agrav = 539,0,-9.80 SI units

Hint 4. Determine which kinematic equation to use


Choose the kinematic equation that makes the solution straightforward, that is, the one that contains the variable you
are solving for and for which all of the other quantities are known.
ANSWER:

v = v 0 + at

2
y = y 0 + v 0 t + (1/2)at

2 2
v = v + 2a(y − y 0 )
0

v0 +v
y− y = ( )t
0 2

ANSWER:

y
max
= 1.75×104 m

Correct

A Flea in Flight

In this problem, you will apply kinematic equations to a jumping flea. Take the magnitude of free-fall acceleration to be 9.80 m/s2
. Ignore air resistance.

Part A
A flea jumps straight up to a maximum height of 0.540 m . What is its initial velocity v 0 as it leaves the ground?

Express your answer in meters per second to three significant figures.

Hint 1. Finding the knowns and unknowns


Take the positive y direction to be upward, the y coordinate of the initial position of the flea to be y0 = 0, and denote
the final height of the flea by y1 , whose value 0.540 m you know. Let t be the duration of the flea's leap to its
maximum height, v 0 its initial velocity, v 1 its final velocity (at maximum height), and ay its (constant) acceleration.
Which of the following quantities is/are known?

Check all that apply.

https://session.masteringphysics.com/myct/assignmentPrintView?assignmentID=6248360 63/86
8/5/2018 Taller de Autoaprendizaje No 1a

Hint 1. The number of known quantities


Typically, you need to know the values of four variables in order to solve any of the kinematic equations,
because they contain five variables each, with the exception of v 1 = v 0 + ay t, which contains only four
variables, in which case you would need to know the values of only three of these variables. Since we may
place the flea at any point of the y axis to begin its jump, we have conveniently assumed that y0 is equal to 0.

Hint 2. What is the flea's velocity at its maximum height?


What is the velocity v 1 of the flea at its maximum height of y1 = 0.540 m ?

Express your answer in meters per second to three significant figures.


ANSWER:

v1 = 0 m/s

ANSWER:

v0

v1

ay

Hint 2. Determine which kinematic equation to use


Decide which kinematic equation makes the solution of this problem easiest. That is, look for an equation that
contains the variable you are solving for and in which all the other variables are known.

ANSWER:

v 1 = v 0 + ay t

1 2
y = y + v0 t + ay t
1 0 2

2 2
v = v + 2ay (y 1 − y 0 )
1 0

v0 +v1
y1 − y0 = ( )t
2

Hint 3. Some algebra help


You have determined that the simplest equation to use is
+ 2ay (y 1 − y 0 ).
2 2
v = v
0 1

To solve for v 0 , you must first subtract the term 2ay (y1 − y0 ) from both sides of the equation, and then take the
square root of both sides. Keep in mind that the acceleration is negative.

ANSWER:

https://session.masteringphysics.com/myct/assignmentPrintView?assignmentID=6248360 64/86
8/5/2018 Taller de Autoaprendizaje No 1a

v0 = 3.25 m/s

Correct

Part B
How long is the flea in the air from the time it jumps to the time it hits the ground?

Express your answer in seconds to three significant figures.

Hint 1. How to approach the problem


One approach is to find the time it takes for the flea to go from the ground to its maximum height, and then find the
time it takes for the flea to fall from its maximum height to the ground. The subsequent hints will guide you through
this approach.

Hint 2. Find the time from the ground to the flea's maximum height
What is the time tup it takes the flea to go from the ground (y0 = 0 m v0 , ) to its maximum height (y1 = 0.540 m ,
v 1 = 0 m)?

Express your answer in seconds to three significant figures.


ANSWER:

t up = 0.332 s

Hint 3. Find the time from the flea's maximum height to the ground
What is the time tdown that it takes for the flea to fall from its maximum height (y0 = 0.540 m , v 0 = 0 m ) to the
ground (y1 = 0 m)?

Express your answer in seconds to three significant figures.


ANSWER:

t down = 0.332 s

ANSWER:

time in air = 0.664 s

https://session.masteringphysics.com/myct/assignmentPrintView?assignmentID=6248360 65/86
8/5/2018 Taller de Autoaprendizaje No 1a

Correct
Notice that the time for the flea to rise to its maximum height is equal to the time it takes for it to fall from that height
back to the ground. This is a general feature of projectile motion (any motion with a = −g) when air resistance is
neglected and the landing point is at the same height as the launch point.

There is also a way to find the total time in the air in one step: just use

1 2
y= y + v0 t + ay t
0 2

and realize that you are looking for the value of t for which y = y
0
.

± The Graph of a Sports Car's Velocity

The graph in the figure shows the velocity v of a sports car as a function of time t.
Use the graph to answer the following questions.

Part A
Find the maximum velocity v max of the car during the ten-second interval depicted in the graph.
Express your answer in meters per second to the nearest integer.

Hint 1. How to approach the problem


Because the graph displays the car's velocity at each moment in time, the maximum velocity of the car can be found
simply by locating the maximum value of the velocity on the graph.

ANSWER:

v max = 55 m/s

Correct

Part B
During which time interval is the acceleration positive?
Indicate the best answer.

https://session.masteringphysics.com/myct/assignmentPrintView?assignmentID=6248360 66/86
8/5/2018 Taller de Autoaprendizaje No 1a

Hint 1. Finding acceleration from the graph


Recall that acceleration is the rate of change of velocity with respect to time. Therefore, on this graph of velocity vs.
time, acceleration is the slope of the graph. Recall that the slope m is defined by m = Δy/Δx for a graph of y vs. x
, or m = Δv/Δt in this case. If the graph is increasing from left to right, then the slope is positive.

ANSWER:

t = 0 s to t = 6 s

t = 0 s to t = 4 s

t = 0 s to t = 10 s

t = 4 s to t = 10 s

t = 2 s to t = 6 s

Correct

Part C
Find the maximum acceleration amax of the car.
Express your answer in meters per second per second to the nearest integer.

Hint 1. How to approach the problem


The car's acceleration is the rate of change of the car's velocity v with respect to time t. In this problem, the car's
velocity is given graphically, so the car's acceleration at a given moment is found from the slope of the v vs. t curve at
that moment. If the v vs. t curve over some time interval is represented by a straight line, the instantaneous
acceleration anywhere in that interval is equal to the slope of the line, that is, to the average acceleration over that
time interval.
To find the maximum acceleration, find the value of the curve's greatest positive slope.

Hint 2. Find the final velocity on the interval with greatest acceleration
The slope of the curve is greatest during the first second of motion. The slope of the graph on this interval is given by
the change in velocity divided by the change in time over the interval from t = 0 to t = 1 . At time t = 0 s, the car's
velocity v(0) is zero. Find the velocity v(1) of the car at time t = 1 s.
Express your answer in meters per second to the nearest integer.

ANSWER:

v(1) = 30 m/s

Correct

ANSWER:

https://session.masteringphysics.com/myct/assignmentPrintView?assignmentID=6248360 67/86
8/5/2018 Taller de Autoaprendizaje No 1a

amax = 30 m/s
2

Correct

Part D
Find the minimum magnitude of the acceleration amin of the car.

Express your answer in meters per second per second to the nearest integer.

Hint 1. How to approach the problem


To find the minimum magnitude of the acceleration of the car, you must find the point where the absolute value of the
slope is smallest.

ANSWER:

amin = 0 m/s
2

Correct

Part E
Find the distance d0,2 traveled by the car between t = 0 s and t = 2 s .

Express your answer in meters to the nearest integer.

Hint 1. How to approach the problem


In this problem, the car's velocity as a function of time is given graphically, so the distance traveled is represented by
the area under the v vs. t graph between t = 0 s and t = 2 s.

Hint 2. Find the distance traveled in the first second


What is the distance d0,1 traveled between t = 0 s and t = 1 s ?

Express your answer in meters.

Hint 1. The area of a triangle


Observe that the region in question is a triangle , whose area is therefore one-half the product of the base and
the height.

https://session.masteringphysics.com/myct/assignmentPrintView?assignmentID=6248360 68/86
8/5/2018 Taller de Autoaprendizaje No 1a

ANSWER:

d_0,1 = 15 m

Hint 3. Find the distance traveled in the second second


What is the distance d1,2 traveled between t = 1 s andt = 2 s ?

Express your answer in meters.

Hint 1. The shape of the region


The region under the graph between 1 and 2 seconds can be seen as consisting of a rectangle and a triangle.

ANSWER:

d1,2 = 40 m

ANSWER:

https://session.masteringphysics.com/myct/assignmentPrintView?assignmentID=6248360 69/86
8/5/2018 Taller de Autoaprendizaje No 1a

d = 55 m

Correct

Prelecture Video: Acceleration

Click Play to watch the video. Answer the ungraded questions in the video and the graded follow-up questions at right.

Part A
Which of the motion diagrams in the figure below best matches the motion of the bungee jumper shown in the video?

ANSWER:

https://session.masteringphysics.com/myct/assignmentPrintView?assignmentID=6248360 70/86
8/5/2018 Taller de Autoaprendizaje No 1a

Correct
Because the video only follows the free fall portion of the jump, the correct motion diagram shows constant
downward acceleration.

Part B
Which of the following best describes the acceleration of a bungee jumper during free fall?

ANSWER:

positive

negative

zero

constant

Correct
Free fall acceleration is constant and has a value of 9.8 m/s2 .

Part C
Three cars drive around a perfectly circular track. The speedometers in the figure below show the speed of each car over the
same 10-s time interval. Which of the cars are accelerating?

ANSWER:

Car A

Cars A and B

Cars A, B, and C

Correct
All three cars are accelerating because the directions of their velocity vectors change constantly as they drive in a
circle. In addition, the speeds of cars A and B are changing, which also indicates acceleration.

Part D

https://session.masteringphysics.com/myct/assignmentPrintView?assignmentID=6248360 71/86
8/5/2018 Taller de Autoaprendizaje No 1a
Four cars undergo acceleration as described by the data in the following table.

Car Initial Velocity (m/s) Final Velocity (m/s) Time period (s)

A 2.0 11.0 3.0

B -5.0 3.0 2.0


C 1.0 -5.0 2.0
D 0.0 25.0 10.0

Rank the accelerations from most positive to most negative.


ANSWER:

Reset Help

most-positive acceleration most-negative acceleration

Car B Car A Car D Car C

The correct ranking cannot be determined.

Correct

The average accelerations, from most positive to most negative, are as follows: (B) 4 m/s2 , (A) 3 m/s2 , (D) 2.5
m/s , and (C) −3 m/s .
2 2

Prelecture Video: Speed and Velocity

Click Play to watch the video. Answer the ungraded questions in the video and the graded follow-up questions at right.

https://session.masteringphysics.com/myct/assignmentPrintView?assignmentID=6248360 72/86
8/5/2018 Taller de Autoaprendizaje No 1a

Part A
Suppose you are hiking along a trail. Make a comparison between the magnitude of your displacement and your distance
traveled.

Check all that apply.

ANSWER:

The magnitude of your displacement can be greater than your distance traveled.

The magnitude of your displacement must be less than your distance traveled.

The magnitude of your displacement can be less than your distance traveled.

The magnitude of your displacement must be equal to your distance traveled.

The magnitude of your displacement can be equal to your distance traveled.

The magnitude of your displacement must be greater than your distance traveled.

Correct
The magnitude of your displacement represents your change in position. That is, the magnitude of your
displacement is how far you are from your starting location along a straight line.

Part B
Suppose a runner completes one lap around a 400-m track in a time of 50 s. Calculate the magnitude of the average
velocity of the runner.
Express your answer in meters per second to the nearest integer.

ANSWER:

0 m/s

Correct

Part C

https://session.masteringphysics.com/myct/assignmentPrintView?assignmentID=6248360 73/86
8/5/2018 Taller de Autoaprendizaje No 1a
Suppose a runner completes one lap around a 400-m track in a time of 50 s. Calculate the average speed of the runner.

Express your answer in meters per second to the nearest integer.


ANSWER:

8 m/s

Correct
Average velocity is defined in terms of displacement, whereas average speed is defined in terms of total distance
traveled.

Part D
Consulting the graph shown in , determine the object's
average velocity over the time interval from 2 to 4 seconds.
Express your answer in meters per second to the nearest
integer.

ANSWER:

5 m/s

Correct
Average velocity can be determined from the slope of a line connecting two points on a position versus time graph.

Exercise 1.4

The density of silver is 10.5 g/cm3 .

Part A
What is this value in kilograms per cubic meter?

ANSWER:

1.05×104
3
kg/m

https://session.masteringphysics.com/myct/assignmentPrintView?assignmentID=6248360 74/86
8/5/2018 Taller de Autoaprendizaje No 1a

Correct

Exercise 1.14

With a wooden ruler you measure the length of a rectangular piece of sheet metal to be 16 mm . You use micrometer calipers to
measure the width of the rectangle and obtain the value 6.04 mm . Give your answers to the following questions to the correct
number of significant figures.

Part A
What is the area of the rectangle?
ANSWER:

A = 97 mm
2

Correct

Part B
What is the ratio of the rectangle's width to its length?
ANSWER:

width
= 0.38
length

Correct

Part C
What is the perimeter of the rectangle?

ANSWER:

P = 44 mm

Correct

Part D
What is the difference between the length and width?

ANSWER:

Δ = 10 mm

https://session.masteringphysics.com/myct/assignmentPrintView?assignmentID=6248360 75/86
8/5/2018 Taller de Autoaprendizaje No 1a

Correct

Part E
What is the ratio of the length to the width?
ANSWER:

length
= 2.6
width

Correct

Exercise 1.18

Part A

How many kernels of corn does it take to fill a 9-L soft drink bottle? (Take that about four kernels fill 1 cm3 .)
Express your answer using two significant figures.
ANSWER:

3.6×104 kernels

Correct

Exercise 2.3

You normally drive on the freeway between San Diego and Los Angeles at an average speed of 105 km/h, and the trip takes 2
h and 20 min. On a Friday afternoon, however, heavy traffic slows you down and you drive the same distance at an average

speed of only 73.5 km/h .

Part A
How much longer does the trip take?
ANSWER:

t = 60.0 min

Correct

Exercise 2.9

https://session.masteringphysics.com/myct/assignmentPrintView?assignmentID=6248360 76/86
8/5/2018 Taller de Autoaprendizaje No 1a
A ball moves in a straight line (the x-axis). The graph in the figure
shows this ball's velocity as a function of time.

Part A
What is the ball's average velocity during the first 2.7 s ?

Express your answer using two significant figures.


ANSWER:

v av = 2.3 m/s

Correct

Part B
What is the ball's average speed during the first 2.7 s ?
Express your answer using two significant figures.

ANSWER:

sav = 2.3 m/s

Correct

Part C
Suppose that the ball moved in such a way that the graph segment after 2.0 s was -3.0 m/s instead of +3.0 m/s. Find the
ball's average velocity during the first 2.7 s in this case.
Express your answer using two significant figures.
ANSWER:

v av = 0.70 m/s

https://session.masteringphysics.com/myct/assignmentPrintView?assignmentID=6248360 77/86
8/5/2018 Taller de Autoaprendizaje No 1a

Correct

Part D
Suppose that the ball moved in such a way that the graph segment after 2.0 s was -3.0 m/s instead of +3.0 m/s. Find the
ball's average speed during the first 2.7 s in this case.

Express your answer using two significant figures.

ANSWER:

sav = 2.3 m/s

Correct

Exercise 2.14

A race car starts from rest and travels east along a straight and level track. For the first 5.0 s of the car's motion, the eastward
component of the car's velocity is given by υx (t) = (0.880 m/s3 )t2 .

Part A
What is the acceleration of the car when υx = 13.1 m/s ?

Express your answer with the appropriate units.


ANSWER:

m
ax = 6.79
s2

Answer Requested

Exercise 2.23

The human body can survive a negative acceleration trauma incident (sudden stop) if the magnitude of the acceleration is less
than 250 m/s2 .

Part A
If you are in an automobile accident with an initial speed of 100 km/h and are stopped by an airbag that inflates from the
dashboard, over what distance must the airbag stop you for you to survive the crash?
ANSWER:

x = 1.54 m

Correct

https://session.masteringphysics.com/myct/assignmentPrintView?assignmentID=6248360 78/86
8/5/2018 Taller de Autoaprendizaje No 1a

Exercise 2.44

A hot-air balloonist, rising vertically with a constant velocity of magnitude v = 5.00 m/s , releases a sandbag at an instant when
the balloon is a height h = 40.0 m above the ground . After it is
released, the sandbag is in free fall. For the questions that follow, take
the origin of the coordinate system used for measuring displacements
to be at the ground, and upward displacements to be positive.

Part A
Compute the position of the sandbag at a time 0.155 s after its release.
ANSWER:

y = 40.7 m

Correct

Part B
Compute the velocity of the sandbag at a time 0.155 s after its release.
ANSWER:

v = 3.48 m/s

Correct

Part C
Compute the position of the sandbag at a time 1.50 s after its release.

ANSWER:

y = 36.5 m

Correct

https://session.masteringphysics.com/myct/assignmentPrintView?assignmentID=6248360 79/86
8/5/2018 Taller de Autoaprendizaje No 1a

Part D
Compute the velocity of the sandbag at a time 1.50 s after its release.

ANSWER:

v = -9.70 m/s

Correct

Part E
How many seconds after its release will the bag strike the ground?
ANSWER:

t = 3.41 s

Correct

Part F
With what magnitude of velocity does it strike?
ANSWER:

v = 28.4 m/s

Correct

Part G
What is the greatest height above the ground that the sandbag reaches?

ANSWER:

y = 41.3 m

Correct

Exercise 2.52

The acceleration of a bus is given by ax (t) = αt , where α = 1.15 m/s3 is a constant.

Part A
If the bus's velocity at time t1 = 1.01 s is 5.02 m/s , what is its velocity at time t2 = 2.19 s ?

https://session.masteringphysics.com/myct/assignmentPrintView?assignmentID=6248360 80/86
8/5/2018 Taller de Autoaprendizaje No 1a
ANSWER:

v = 7.19 m/s

Correct

Part B
If the bus's position at time t1 = 1.01 s is 5.99 m , what is its position at time t2 = 2.19 s ?
ANSWER:

x = 13.0 m

Correct

PhET Tutorial: Moving Man

Learning Goal:
To understand the relationships between position, velocity, and acceleration.

For this tutorial, use the PhET simulation The Moving Man. This simulation allows you to drag a person back and forth and look
at the resulting position, velocity, and acceleration. You can also enter a position as a function of time mathematically and look at
the resulting motion.

Start the simulation. When you click the simulation link, you may be asked whether to run, open, or save the file. Choose to run
or open it.

Under the Charts tab you can click and drag the person left and right, or enter a numeric value in the boxes on the left panel to
see plots for the person’s position, velocity, and acceleration as a function of time. Click the Play button to start a simulation and
the Pause button to stop a simulation. You can also watch a playback by selecting the Playback radio button instead of the
default Record radio button. You can click Clear to remove the current plot while maintaining your settings for position, velocity
and acceleration or click Reset All to start over. In the Playback mode, the grey bar can be dragged over the plot to any value in
time, and the digital readouts will show the corresponding values of the position, velocity, and acceleration.

Under the Special Features menu, the Expression Evaluator option produces a second window in which you can mathematically
type in any function for the position as a function of time, x(t). After typing in a function, click the Play button to start the
simulation.
https://session.masteringphysics.com/myct/assignmentPrintView?assignmentID=6248360 81/86
8/5/2018 Taller de Autoaprendizaje No 1a
To zoom in vertically, click any of the three + buttons to the top right of each plot. To zoom in horizontally, click the + button to
the bottom right of the acceleration plot.
Feel free to play around with the simulation. When you are done, click Reset All on the Charts tab before beginning Part A.

Part A
First, you will focus on the relationship between velocity and position. Recall that velocity is the rate of change of position (
v x = dx/dt). This means that the velocity is equal to the slope of the Position vs. Time graph.

Move the person to the position x = −6 m or enter –6.00 in the position box. If you dragged the person to position, click the
Pause button and then the Clear button. Next, drag the person to the right to roughly x = 6 m and reverse his direction,
returning him to the original position, at x = −6 m . Move the person relatively quickly, about a few seconds for the round
trip. Your plots should look something like those shown below.

Look at the Position vs. Time and Velocity vs. Time plots. What is the person's velocity when his position is at its maximum
value (around 6 m )?
ANSWER:

negative.
The person's velocity is positive.

zero.

Correct
When the person’s position is a maximum, the slope of the position with respect to time is zero, so dx/dt = 0.
However, due to the person’s acceleration, the velocity does not remain zero; he eventually moves to the left.

Part B

Acceleration is the rate of change of the velocity, ax = dv x /dt , so it is the slope of the Velocity vs. Time graph.

Because it is difficult to drag the person in a consistent and reproducible way, use the Expression Evaluator under the
Special Features menu for this question.

https://session.masteringphysics.com/myct/assignmentPrintView?assignmentID=6248360 82/86
8/5/2018 Taller de Autoaprendizaje No 1a

Click Reset All and type in the function x(t) = 8 ∗ t − 2 ∗ t ∗ t in the Expression Evaluator. Click the Play button and let
the simulation run roughly 5 simulation seconds before pressing the Pause button. Use the zoom buttons to adjust the plots
so they fit in the screen. You should see a plot similar to what you got in the previous question, but much smoother.

Look at the Position vs. Time, Velocity vs. Time, and Acceleration vs. Time plots.

Hint 1. How to approach the problem


In Playback mode, use the grey vertical bar. Slide the bar until the value x = 8 m is displayed in the position box on
the left panel. What are the values of velocity and acceleration when x = 8 m?

ANSWER:

the velocity is zero but the acceleration is negative.

When the person is 8 m to the right of the origin, both the velocity and the acceleration are zero.

the velocity is zero but the acceleration is positive.

both the velocity and the acceleration are nonzero.

Correct
At x = 8 m, the person turns to go back in the opposite direction. His velocity is zero, but his acceleration is
negative since the velocity is decreasing with time. This is similar to throwing a ball straight up into the air; at its
highest point, the velocity is zero but the acceleration is still directed downward.

Part C

Keep the function x(t) = 8∗t−2∗t∗t in the Expression Evaluator. What is the value of the person’s acceleration a at
t = 2 s?

Hint 1. How to approach the problem


Use the grey vertical bar. Slide the bar until it coincides with t = 2s on the horizontal axis. What is the value
displayed in the acceleration box on the left panel?

ANSWER:

2
4 m/s

0
ax =
2
−2 m/s

2
−4 m/s

https://session.masteringphysics.com/myct/assignmentPrintView?assignmentID=6248360 83/86
8/5/2018 Taller de Autoaprendizaje No 1a

Correct
This is an example of one-dimensional motion with constant acceleration. The position of an object undergoing this
type of motion obeys the kinematic equation x(t) = x0 + v x,0 t + 1/2 ax t2 . In this case, the initial velocity is
v x,0 = 8 m/s and the acceleration is ax = −4 m/s (since 1/2 ax = −2 m/s ).
2 2

Part D

In the previous question, the person had an initial velocity of 8 m/s and a constant acceleration of −4 m/s2 . How would the
maximum distance he travels to the right of the origin change if instead his initial velocity were doubled (v x,0 = 16 m/s)?

Hint 1. How to approach the problem


Go to the Introduction tab to run the simulation using the new initial velocity v x,0 = 16 m/s and the same
acceleration of −4 m/s , and read the value for position when the velocity equals zero. Remember to remove the
2

walls from the simulation by clicking on the red close button on the walls. In Playback mode the simulation can be
run slowly and paused when the velocity is zero.

Or, mathematically, determine how long it takes for the person to stop and use this value of time in the equation for
x(t) .

ANSWER:

The maximum distance would double.

The maximum distance would not change.

The maximum distance would increase by a factor of four.

Correct
Because it takes twice as much time to momentarily stop, and because his average velocity will be twice as fast,
the distance he travels will be four times greater. Using the kinematic equation,
2
x(4 s) = (16 m/s) × (4 s) − (1/2) × (4 m/s ) × (4 s) = 32 m.
2

Part E

Now, assume that the position is given by the equation x(t) = 4t


3
.

Enter this function in the Expression Evaluator as x(t) = 4 ∗ t ∗ t ∗ t. Run the simulation by clicking the Play button in the
Record mode for roughly three simulated seconds and then click the Pause button. Now take a look at the graphs. You will
have to zoom in horizontally (bottom right), so that your range covers two seconds.

Which of the following statements is true?

ANSWER:

https://session.masteringphysics.com/myct/assignmentPrintView?assignmentID=6248360 84/86
8/5/2018 Taller de Autoaprendizaje No 1a

The acceleration is constant in time.

The acceleration is increasing at a constant rate.

The velocity is increasing at a constant rate.

The position is increasing at a constant rate.

Correct
The graph showing Acceleration vs. Time is a straight line that is not horizontal.

Part F
What is the position of the person when t = 1s ?
Express your answer numerically in meters to one significant figure.

Hint 1. How to approach the problem


You can solve this mathematically using the expression input for position as a function of time, x(t) = 4∗t∗t∗t .

Alternatively, you can put the simulation in Playback mode and drag the grey box/bar to t = 1s , then read the
approximate value for position.

ANSWER:

4 m

Correct
3
Notice that since the position is given by x = 4t
3
, when the time is t = 1s , the position is x = 4(1) m = 4 m.

Part G
What is the velocity of the person when t = 1s ?

Express your answer numerically in meters per second to two significant figures.

Hint 1. How to approach the problem


The velocity is the first derivative of position with respect to time, v x = dx/dt . You can take the derivative of the
expression of x(t) and evaluate when t = 1 s.

Alternatively, you can put the simulation in Playback mode and drag the grey box/bar to t = 1s , then read the
approximate value for velocity.

ANSWER:

https://session.masteringphysics.com/myct/assignmentPrintView?assignmentID=6248360 85/86
8/5/2018 Taller de Autoaprendizaje No 1a

12 m/s

Correct

Notice that since the position is given by x(t) = 4t3 , the velocity, which is the first derivative of position with
respect to time, is given by v x = dx/dt = 12t2 . So when t = 1 s, v = 12 m/s.

Part H
What is the acceleration of the person when t = 1s ?
Express your answer numerically in meters per second squared to two significant figures.

Hint 1. How to approach the problem


The acceleration is the first derivative of velocity with respect to time, ax = dv x /dt or the second derivative of
position with respect to time, ax = d 2 x/dt2 . You can take the derivative of the expression of v x (t) and evaluate
when t = 1 s.

Alternatively, you can put the simulation in Playback mode and drag the grey box/bar to t = 1s , then read the
approximate value for acceleration.

ANSWER:

24 m/s
2

Correct
Notice that since the position is given by x = 4t3 , the acceleration, which is the first derivative of velocity with
respect to time, is given by ax = dv x /dt = d(12t2 )/dx = 24t. So when t = 1 s, ax = 24 m/s2 . Notice also
that the acceleration is proportional to time, which explains why it is increasing at a constant rate (as discovered in
part G).

PhET Interactive Simulations


University of Colorado
http://phet.colorado.edu
Score Summary:
Your score on this assignment is 91.4%.
You received 30.17 out of a possible total of 33 points.

https://session.masteringphysics.com/myct/assignmentPrintView?assignmentID=6248360 86/86

Você também pode gostar